EMG Qbank Flashcards

1
Q

A patient’s right upper limb EMG/NCS results are as follows. Routine median, ulnar, and radial sensory nerve conduction studies are normal except for reduced amplitude of ulnar SNAP to digit 5. EMG reveals decreased recruitment and polyphasic potentials in triceps, extensor indicis proprius, flexor digitorum profundus to digits 2-5, flexor carpi ulnaris, abductor pollicis brevis, and abductor digiti minimi, but normal activity in supraspinatus, deltoid, biceps, brachioradialis, and flexor carpi radialis. Which of the following is the most likely diagnosis?

A

This is a lower trunk plexopathy. Radiculopathy would spare the SNAPs, which is affected here for the ulnar nerve. Middle trunk plexopathy would affect the flexor carpi radialis, which is spared here

How well did you know this?
1
Not at all
2
3
4
5
Perfectly
2
Q

During nerve conduction studies, why do we refer to the CMAP as the “compound” muscle action potential?

A

Because the CMAP records an electrical summation of all muscle fibers of a single muscle contraction

How well did you know this?
1
Not at all
2
3
4
5
Perfectly
3
Q

A previously healthy 53 year-old female presents with gradual onset weakness, worse with any type of exercise, climbing stairs, and worse as the day progresses. She does not smoke, drink alcohol, or use illicit drugs. She notices associated double vision at times, worse as the day progresses. She denies trauma or bowel/bladder abnormalities. Which of the following etiologies is most likely responsible for her symptoms?

A

Presence of antibodies against postsynaptic acetylcholine receptors
- myasthenia gravis (MG)

How well did you know this?
1
Not at all
2
3
4
5
Perfectly
4
Q

Which of the following is the main problem with performing the Phalen test with the elbows flexed instead of extended?

A

A positive test is less informative
-When performing the Phalen test (flexing the wrists, holding this posture, and eliciting hand numbness/tingling due to median nerve compression within the carpal tunnel), it is best performed with the patient’s elbows extended. Flexing the elbows stretches the ulnar nerve and may irritate an already irritated ulnar nerve, thus causing hand numbness and tingling NOT due to median nerve compression. Thus, a positive test in that case would not allow you to narrow it down between ulnar neuropathy and median neuropathy - it is less informative. The other answer choices are fictional.

How well did you know this?
1
Not at all
2
3
4
5
Perfectly
5
Q

Which of the following is the most likely cause of a patient’s lumbosacral plexopathy?

A

diabetes is the most likely to cause lumbosacral plexopathy.

How well did you know this?
1
Not at all
2
3
4
5
Perfectly
6
Q

During nerve conduction studies, sensory nerves are typically recorded in which way?

A

Antidromically
- SNAPs are typically recorded antidromically to obtain a clearer, “louder” signal from the nerve.

How well did you know this?
1
Not at all
2
3
4
5
Perfectly
7
Q

A 60 year-old female presents with proximal hip and shoulder weakness, periorbital erythematous rash, and erythematous papules over her metacarpophalangeal (MCP) and interphalangeal (IP) joints of the hands. Which of the following EMG findings is most likely present in this patient?

A

Early/increased recruitment
-patient presents with classic dermatomyositis (proximal muscle weakness with heliotrope rash and Gottron papules). This is a myopathic disease; thus, the EMG will show signs of myopathy (short-duration, small amplitude MUAPs with early/increased recruitment). Decreased recruitment is found in nerve disorders, not muscle disease. Giant MUAPs are found in post-polio syndrome. Decreased SNAP amplitudes can be found in any sensory nerve disease that affects axons.

How well did you know this?
1
Not at all
2
3
4
5
Perfectly
8
Q

Which of the following structures forms the lateral border of Guyon’s canal?

A

lateral border : hook of the Hamate
medial border : pisiform\

How well did you know this?
1
Not at all
2
3
4
5
Perfectly
9
Q

An EMG potential is described as polyphasic if it crosses the baseline a minimum of how many times?

A

Polyphasicity is defined as 5 or more phases. Phases are defined as the number of baseline crossings + 1. Thus, polyphasic potentials are defined as a baseline crossing of 4 or more times, which would equate to 5 or more phases.

How well did you know this?
1
Not at all
2
3
4
5
Perfectly
10
Q

Lowering the high frequency filter during NCS will cause which of the following changes?

A

Prolonged peak latency, prolonged onset latency

How well did you know this?
1
Not at all
2
3
4
5
Perfectly
11
Q

When a muscle recruits its first motor unit during initial contraction, it tends to fire at _____ Hz, and subsequent motor units are recruited for every ______ Hz increase in firing rate.

A

5, 5
-The initial motor unit is recruited at 5 Hz; every 5 Hz an additional motor unit is added to the recruitment pattern. Thus, your EMG screen should at some point show you 4 motor units firing at once with gradually increased muscle contraction by the patient: these units will be firing at 20, 15, 10, 5 Hz respectively.

How well did you know this?
1
Not at all
2
3
4
5
Perfectly
12
Q

Neurapraxia can be defined as which of the following?

A

Focal pressure on a nerve, leading to focal demyelination and conduction block
-This can become remyelinated over 2-3 weeks,

How well did you know this?
1
Not at all
2
3
4
5
Perfectly
13
Q

You are performing an EMG/NCS on a patient. The sensory nerve action potential (SNAP) to digit 2 and compound motor action potential (CMAP) to the abductor pollicis brevis are normal. The SNAP to digit 5 reveals prolonged latency. The CMAP to the abductor digiti minimi is normal. Needle EMG reveals no abnormalities. Which of the following is the most likely diagnosis?

A

Mild ulnar neuropathy

How well did you know this?
1
Not at all
2
3
4
5
Perfectly
14
Q

Which of the following muscles is most likely to develop weakness following a proximal humerus surgical neck fracture?

A

axillary nerve is at greatest risk of injury following a surgical neck fracture.
The only muscle listed that is innervated by the axillary nerve is teres minor. Deltoid (not listed) would have also been an acceptable answer.
Supraspinatus and infraspinatus - suprascapular nerve. ECRB - radial nerve.

How well did you know this?
1
Not at all
2
3
4
5
Perfectly
15
Q

You are reviewing EMG/NCS results of a patient. You notice prolonged latency, decreased conduction velocity, increased temporal dispersion, and normal amplitude on NCS. There is decreased recruitment on EMG. Which of the following conclusions is most likely correct?

A
How well did you know this?
1
Not at all
2
3
4
5
Perfectly
16
Q

A patient’s left upper limb EMG/NCS results are as follows. Sensory nerve action potential (SNAP) of the ulnar nerve to digit 5 exhibits decreased amplitude. EMG reveals decreased recruitment and 3+ fibrillations in pectoralis major, flexor digitorum superficialis, flexor carpi ulnaris, abductor pollicis brevis, and first dorsal interosseous, but normal activity in deltoid, biceps, brachioradialis, triceps, and extensor indicis proprius. Which of the following EMG/NCS findings would you also expect to find in this patient?

A

Decreased amplitude of medial antebrachial cutaneous nerve SNAP

How well did you know this?
1
Not at all
2
3
4
5
Perfectly
17
Q

You are reviewing EMG/NCS results of the right lower limb. The superficial fibular sensory nerve action potential (SNAP) is abnormal while the sural SNAP is normal. Needle EMG reveals decreased recruitment and +1 fibrillations in the fibularis longus and normal activity in tibialis anterior, extensor digitorum brevis, abductor hallucis, gastrocnemius, medial hamstrings, rectus femoris, tensor fascia lata, and lumbar paraspinals. Which of the following is the most likely diagnosis?

A

Superficial fibular neuropathy
- Sciatic neuropathy would involve the gastrocnemius and abductor hallucis, as well as tibialis anterior and extensor digitorum brevis theoretically, which are all spared here. Deep fibular neuropathy would spare the fibularis longus and affect the tibialis anterior and extensor digitorum brevis.

How well did you know this?
1
Not at all
2
3
4
5
Perfectly
18
Q

During a blink reflex study, when stimulating the left trigeminal nerve, you detect a single left eye blink response. This is known as which of the following?

A

The quick, ipsilateral blink response is called the R1 response, and we record it from the orbicularis oculi muscle ipsilaterally to the side of stimulation.

How well did you know this?
1
Not at all
2
3
4
5
Perfectly
19
Q

During a nerve conduction study you realize that the patient’s limb being studied is cold. Which of the following effects will this have on the waveform?

A

A cold limb will result in increased amplitude, prolonged latency (slow conduction velocity), and increased duration. When the limb is cold, channels stay open longer, causing a larger, heftier amplitude, and longer time of depolarizing.

How well did you know this?
1
Not at all
2
3
4
5
Perfectly
20
Q

During an exam, perhaps much like this one, when a question stem mentions EMG findings of painless myokymia in C5-C6/upper trunk muscles, you will astutely recall that the most likely etiology of these findings is which of the following?

A

Radiation plexopathy is classically the cause of myokymia found on EMG, which classically affects C5-C6/upper trunk muscles, and is usually a painless finding. Lower trunk plexopathy with pain is concerning for pancoast (lung) tumor compressing the lower trunk of the brachial plexus. Myotonia congenita will demonstrate myotonic discharges (divebomber sound).

How well did you know this?
1
Not at all
2
3
4
5
Perfectly
21
Q

Demyelination will most likely result in which of the following nerve conduction study (NCS) results?

A

Normal amplitude, prolonged latency, increased temporal dispersion
-Demyelination is the stripping of the myelin sheath around axons. As myelin increases conduction velocity of axons (decreasing their latency), removal of this myelin will cause slowed conduction velocity, prolonged latency (it takes a longer time for the action potential to reach the recording electrode), and increased temporal dispersion. Temporal dispersion is the widening of the SNAP or CMAP due to impulses reaching the recording electrode at widely varying times (due to demyelination decreasing the uniformity of signal transmission along a nerve’s axons). Amplitude decrease occurs with axonal loss. Amplitude increase occurs in cold limbs.

How well did you know this?
1
Not at all
2
3
4
5
Perfectly
22
Q

A stretch injury to a nerve that leads to axon loss with intact epineurium is also known as which of the following?

A

-This question describes axonotmesis. Even though the axons have died due to crush/stretch injury, the epineurium is still intact, which will serve as a guide path for the axons to regenerate along and ultimately find their target muscle fibers again.

How well did you know this?
1
Not at all
2
3
4
5
Perfectly
23
Q

Which of the following correctly states the innervation of the flexor digitorum superficialis of the forearm?

A

“C7, C8, middle and lower trunk, medial and lateral cord, median nerve” correctly states the FDS innervation.

How well did you know this?
1
Not at all
2
3
4
5
Perfectly
24
Q

A patient presents with left foot drop. On EMG/NCS, you note normal superficial fibular and sural nerve SNAPs (sensory nerve action potentials). EMG reveals 3+ positive sharp waves and decreased recruitment in the tibialis anterior (TA), extensor digitorum longus (EDL), extensor hallucis longus (EHL), and extensor digitorum brevis (EDB). EMG of the fibularis longus, abductor hallucis (AH), medial gastrocnemius, hamstrings, rectus femoris, thigh adductors, tensor fascia lata, and lumbar paraspinals is normal. Which of the following is the most likely diagnosis?

A

Deep fibular neuropathy is the best answer. Superficial fibular neuropathy would affect fibularis longus and superficial fibular SNAP, and spare all the affected muscles in this question. Distal sciatic neuropathy would affect gastrocnemius and AH, as well as fibularis longus, and potentially superficial fibular SNAP. L4 radiculopathy would indeed show normal SNAPs, but would also be expected to affect other muscles receiving L4 innervation, such as rectus femoris, hamstrings, and thigh adductors.

How well did you know this?
1
Not at all
2
3
4
5
Perfectly
25
Q

You are performing an EMG on a patient with hand numbness. You note prolonged latency of the sensory nerve action potential (SNAP) to digit 5. The compound motor action potential (CMAP) to the abductor digiti minimi (ADM) is normal when stimulating at the wrist, but reduced when stimulating just below the elbow. What is the next most appropriate step?

A

This question is a classic example of discovering a Martin-Gruber Anastomosis (MGA) in a patient. This is a normal anatomic variant by which median nerve fibers cross over into the ulnar nerve somewhere in the forearm to supply innervation to the ulnar nerve muscles of the hand. This patient seems like they have a conduction block of the ulnar nerve somewhere in the forearm - however, this is rare, and MGA is much more common and likely. In case of MGA, record over the abductor digiti minimi (as you are doing already in this study) and stimulate the median nerve at the elbow, and see if the amplitude is repaired in your “conduction block”. If all of the amplitude is repaired, then you have MGA, and not conduction block of the ulnar nerve. This is the most appropriate next step here, as conduction block of the ulnar nerve in the middle and distal forearm is rare.

How well did you know this?
1
Not at all
2
3
4
5
Perfectly
26
Q

While performing a nerve conduction study (NCS) you apply current to the nerve and detect a small amplitude. You then move distally on the limb and apply current again to the nerve. To your surprise, the amplitude is normal. What is the most likely explanation for this finding?

A

Partial conduction block

How well did you know this?
1
Not at all
2
3
4
5
Perfectly
27
Q

You are performing serial EMGs on a patient. Over time you notice that while he used to demonstrate polyphasic motor unit action potentials (MUAPs) in a particular muscle, on his most recent EMG these polyphasic potentials have been replaced by larger, non-polyphasic potentials. What is the most reasonable explanation for this finding?

A

Polyphasic potentials represent ongoing reinnervation due to collateral sprouting of existing axons. The polyphasic property arises due to non-uniform myelination of these collateral sprouts during this early reinnervation stage. Once myelination is complete, the motor unit’s branches all conduct the action potential at essentially the same speed, producing a nice, uniform “roller coaster” bump (uniform-appearing MUAP) rather than the polyphasic, serrated potential seen in unmyelinated collateral sprouts.

How well did you know this?
1
Not at all
2
3
4
5
Perfectly
28
Q

A patient’s right lower limb EMG/NCS results are as follows. Routine fibular and tibial motor/sensory nerve conduction studies are normal. EMG reveals decreased recruitment and polyphasic potentials in tibialis anterior, extensor hallucis longus, short head of biceps femoris, tensor fascia lata, fibularis longus, and tibialis posterior, but normal activity in vastus medialis, adductor longus, gastrocnemius, and abductor hallucis. Which of the following is the most likely diagnosis?

A

L5 radiculopathy with reinnervation is the best answer. All L5-innervated muscles are affected (with polyphasic potentials, indicating early reinnervation taking place), but all muscles that do not share any L5 innervation are spared.

How well did you know this?
1
Not at all
2
3
4
5
Perfectly
29
Q

Which of the following types of dermatomyositis is associated with cancer?

A

3

How well did you know this?
1
Not at all
2
3
4
5
Perfectly
30
Q

What does a polyphasic potential represent during an EMG?

A

Polyphasic potentials represent ongoing reinnervation due to collateral sprouting of existing axons. The polyphasic property arises due to non-uniform myelination of these collateral sprouts during this early reinnervation stage.
>

How well did you know this?
1
Not at all
2
3
4
5
Perfectly
31
Q

During an EMG, with the muscle at rest, you decide to advance your needle until you hear a “seashell” sound. What does this sound represent?

A

Miniature endplate potentials
- The “seashell” sound is the sound of miniature endplate potentials (MEPPs), which are heard when the needle is very close to endplates, which is a painful needle location for the patient.

How well did you know this?
1
Not at all
2
3
4
5
Perfectly
32
Q

The sternocleidomastoid (SCM) shares its innervation with which of the following muscles?

A

The SCM and trapezius are innervated by C3, C4; spinal accessory nerve. The rhomboids are innervated by C4, C5; dorsal scapular nerve. It’s not important for you to know splenius capitis’ innervation, but it’s not the spinal accessory nerve.

How well did you know this?
1
Not at all
2
3
4
5
Perfectly
33
Q

While examining a patient you notice that their right scapula sits more medially than the left scapula. Needle EMG would most likely reveal abnormalities of musculature belonging to which nerve?

A

The two nerves associated with winged scapula (this patient has a medially winged scapula) are the long thoracic nerve, which innervates the serratus anterior and whose injury can cause a medially winged scapula, and the spinal accessory nerve, which innervates the trapezius and whose injury can cause a laterally winged scapula. The dorsal scapular nerve innervates the rhomboids, and the thoracodorsal nerve innervates the latissimus dorsi.

How well did you know this?
1
Not at all
2
3
4
5
Perfectly
34
Q

You are performing an EMG/NCS on a patient. NCS reveals abnormal sensory nerve action potential (SNAP) to digit 5. Dorsal ulnar cutaneous nerve (DUC) SNAP latency is prolonged. Which of the following is the most likely diagnosis?

A
How well did you know this?
1
Not at all
2
3
4
5
Perfectly
35
Q

Complex repetitive discharges occur via which of the following mechanisms?

A

A CRD is an involuntary discharge that is very wide and serrated (complex in appearance, and repetitive in firing) and occurs due to a motor unit being denervated and then reinnervated by another motor neuron, which itself then becomes denervated. Ephaptic transmission is the process by which these muscle fibers all fire regularly together. CRDs are seen in chronic radiculopathy, anterior horn cell disease, and some normal patients.

How well did you know this?
1
Not at all
2
3
4
5
Perfectly
36
Q

A 45 year-old female presents with 3 months of right foot drop and leg pain. On exam, tinel at the fibular head is positive. Nerve conduction studies of the right lower limb (NCS) reveal decreased amplitude of sural and superficial fibular SNAPs (sensory nerve action potential) and decreased amplitude of extensor digitorum brevis (EDB) CMAP (compound muscle action potential). EMG reveals decreased recruitment of tibialis anterior (TA), EDB, abductor hallucis (AH), fibularis longus, and semitendinosus. EMG of lumbar paraspinals, gluteus maximus, tensor fascia lata (TFL), rectus femoris, and thigh adductors is normal. Which of the following is the most likely diagnosis?

A

Proximal sciatic neuropathy is the best choice, as all sciatic nerve-innervated muscles (some hamstrings and the gastroc/soleus are untested) are affected on EMG, as well as both tibial and fibular SNAPs being affected as well, indicating that most parsimonious explanation is a proximal sciatic neuropathy. A distal sciatic neuropathy, such as at the knee, would spare the hamstrings, which are affected in this case. A radiculopathy would show normal SNAPs (abnormal in this case). Common fibular neuropathy would spare the hamstrings and the abductor hallucis, both of which are affected in this case.

How well did you know this?
1
Not at all
2
3
4
5
Perfectly
37
Q

A patient’s right lower limb EMG/NCS results are as follows. Routine fibular and tibial motor/sensory nerve conduction studies are normal. EMG reveals decreased recruitment and polyphasic potentials in tensor fascia lata, rectus femoris, adductor longus, semimembranosus, and tibialis anterior, but normal activity in short head of biceps femoris, fibularis longus, gastrocnemius, and abductor hallucis. Which of the following is the most likely diagnosis?

A

All muscles receiving some degree of L4 innervation are affected here, but all those without L4 innervation are spared. Note that the short head of biceps femoris is innervated by L5-S1, common fibular division of sciatic nerve, thus differentiating it from the medial hamstrings and long head of biceps femoris, which have L4-L5-S1, tibial division of sciatic nerve innervation. Note also that typically an S1 radiculopathy question will refer to an abnormal H-reflex as partial evidence of an S1 radiculopathic process.

How well did you know this?
1
Not at all
2
3
4
5
Perfectly
38
Q

You are performing an EMG on a patient with hand numbness. You note prolonged latency of the sensory nerve action potential (SNAP) to digit 5. The compound motor action potential (CMAP) to the abductor digiti minimi (ADM) is normal when stimulating at the wrist, but reduced when stimulating above the elbow and near the axilla. What is the next best step?

A

This question demonstrates low CMAP amplitude when stimulating proximally, but normal when stimulating distally, indicating that conduction block is taking place somewhere between the above-elbow stimulation site and the wrist. The next best step would be to stimulate below the elbow and see if the amplitude is normal again. If normal, then you have confirmed a conduction block somewhere at the elbow (between the above-elbow site and below-elbow site). If abnormal, then the conduction block is somewhere in the forearm between the below-elbow site and the wrist. If abnormal, again, this can also be a Martin-Gruber Anastomosis, the normal anatomic variant by which median nerve fibers cross over into the ulnar nerve somewhere in the forearm to supply innervation to the ulnar nerve muscles of the hand. In case of MGA, record over the abductor digiti minimi (as you are doing) and stimulate the median nerve at the elbow, and see if the amplitude is repaired in your “conduction block”. If all of the amplitude is repaired, then you have MGA, and not conduction block of the ulnar nerve

How well did you know this?
1
Not at all
2
3
4
5
Perfectly
39
Q

When needling a patient’s biceps brachii during an EMG study, you detect an involuntary, abrupt, regular signal that sounds like soldiers marching. What is the most likely diagnosis?

A

Radiation plexopathy

How well did you know this?
1
Not at all
2
3
4
5
Perfectly
40
Q

Which of the following muscles is NOT innervated by the posterior interosseous nerve?

A

The ECRL is innervated by the radial nerve, not the posterior interosseous nerve (which arises from the radial nerve).

How well did you know this?
1
Not at all
2
3
4
5
Perfectly
41
Q

During nerve conduction studies, the optimal lower limb temperature is which of the following?

A

The optimal lower limb temperature during NCS is 30 degrees Celsius.

How well did you know this?
1
Not at all
2
3
4
5
Perfectly
42
Q

The nerve that provides cutaneous sensation to the medial leg below the knee arises via which of the following ways?

A

The saphenous nerve is described here, and is the continuation of the femoral nerve after it has innervated all its muscles.

How well did you know this?
1
Not at all
2
3
4
5
Perfectly
43
Q

During a patient examination for left shoulder pain you notice that the patient’s left scapula sits more medially than the right scapula. The nerve implicated in this patient’s condition receives its innervation from which trunk of the brachial plexus?

A

The long thoracic nerve innervates the serratus anterior. Injury to the long thoracic nerve can cause a medially winged scapula, as this patient demonstrates. The serratus anterior innervation is C5, C6, C7, long thoracic nerve. Thus, its innervation is pre-plexus, and does not have any trunks or cords that contribute to it.

How well did you know this?
1
Not at all
2
3
4
5
Perfectly
44
Q

A 62 year-old male with a past medical history of hypertension presents with progressive onset weakness and swallowing difficulties. He is a retired professional football player. He denies numbness/tingling, bowel/bladder dysfunction, or a family history of this problem. EMG/NCS reveals normal sensory nerve action potentials (SNAPs) and compound muscle action potentials (CMAPs) in 3 limbs. Needle exam reveals long-duration, large amplitude motor unit action potentials (MUAPs) with decreased recruitment and 2+ fibrillations in 3 limbs. Which of the following treatments is most likely appropriate?

A

This patient’s presentation and EMG findings are classic for ALS (amyotrophic lateral sclerosis). This is the death of anterior horn cells due to a not-yet clearly defined cause. It has been associated with a professional athletics history. Recall that anterior horn cells are entirely separate from sensory neurons, and, thus, the SNAPs of these patients will be normal. CMAPs can be normal vs. abnormal in these patients. EMG shows a “neuropathic” pattern of MUAPs as described. Riluzole has been shown to prolong survival in ALS patients. Rehabilitation with submaximal exercise is recommended. ALS carries a poor prognosis, and most patients experience progressive disability and death within a few years of onset. Nusinersen is indicated in patients with Spinal Muscular Atrophy.

How well did you know this?
1
Not at all
2
3
4
5
Perfectly
45
Q

The medial antebrachial cutaneous nerve (MAC) arises in which of the following ways?

A

The MAC arises as a branch of the medial cord of the brachial plexus.

How well did you know this?
1
Not at all
2
3
4
5
Perfectly
46
Q

In Erb palsy, which of the following EMG/nerve conduction study findings is most likely to be discovered? SNAP: sensory nerve action potential.

A

Abnormal lateral antebrachial cutaneous nerve SNAP
-Erb palsy is a C5, C6/upper trunk brachial plexopathy commonly caused by trauma as an adult or obstetrical trauma as an infant due to traction forces on these roots. Because the C5 and C6 roots are injured, changes can be expected to be found in any downstream SNAPs, CMAPs, or EMG needling of C5, C6 muscles. The Waiter’s Tip position is classic for this: arm is adducted, internally rotated, pronated, wrist flexed, all due to C5/C6 muscle weakness. There is sensory loss over the lateral arm and dorsolateral forearm. The musculocutaneous nerve (C5, C6, upper trunk) terminates as the lateral antebrachial cutaneous nerve (LAC); thus, if the C5,C6/upper trunk is injured, an abnormal LAC SNAP may be detected. The median sensory fibers to the thumb involve C5/C6 fibers; thus, this median SNAP would be abnormal on NCS. The extensor indicis proprius (EIP) does not contain C5, C6, or upper trunk innervation, and thus would show normal recruitment in this case. The medial antebrachial cutaneous (MAC) SNAP would be normal as well, as it arises from the lower trunk of the brachial plexus, which itself arises from C8 and T1 nerve roots.

How well did you know this?
1
Not at all
2
3
4
5
Perfectly
47
Q

Which of the following muscles does not receive its innervation from the lateral cord of the brachial plexus?

A

Flexor carpi ulnaris
-Biceps brachii, flexor carpi radialis, and pronator teres all receive lateral cord innervation. Flexor carpi ulnaris receives medial cord innervation.

How well did you know this?
1
Not at all
2
3
4
5
Perfectly
48
Q

A 47 year-old male presents to your musculoskeletal clinic with complaints of 3 months of left shoulder pain. He works in the post office handling heavy packages. On exam, you note no pain with empty can, and no pain with resisted external or internal rotation of the shoulder. When abducting the arm 90 degrees and flexing the elbow 90 degrees, and then performing resisted external rotation of the shoulder, this reproduces the patient’s pain. What is the innervation of the muscle being tested?

A

The physical exam maneuver described is how one isolates and tests the teres minor (one could also argue infraspinatus, which is difficult to isolate from teres minor on physical examination), which is innervated by C5,C6, upper trunk, posterior cord, axillary nerve. The dorsal scapular nerve innervates the levator scapula, rhomboid minor, and rhomboid major, whose physical examination tests would involve scapular elevation and/or medial translation. The upper and lower subscapular nerves innervate the subscapularis and teres major (lower subscapular nerve → teres major), tested differently from how this question describes.

How well did you know this?
1
Not at all
2
3
4
5
Perfectly
49
Q

When performing an ultrasound-guided corticosteroid injection into the 1st extensor compartment, which of the following is the most likely potential complication?

A

The superficial radial nerve runs right along the 1st extensor compartment, and is susceptible to stretching/compression/crush if not identified and accounted for prior to advancing the needle towards the 1st extensor compartment (which contains APL and EPB tendons). This procedure is commonly done for De Quervain Tenosynovitis.

How well did you know this?
1
Not at all
2
3
4
5
Perfectly
50
Q

A patient’s left upper limb EMG/NCS results are as follows. Routine median, ulnar, and radial sensory nerve conduction studies are normal. EMG reveals decreased recruitment and 4+ fibrillations in triceps, extensor indicis proprius, abductor pollicis brevis, and first dorsal interosseous, but normal resting activity and recruitment in deltoid, biceps, brachioradialis, and pronator teres. Which of the following is the most likely diagnosis?

A

C8 radiculopathy

How well did you know this?
1
Not at all
2
3
4
5
Perfectly
51
Q

A patient’s right upper limb EMG/NCS results are as follows. Routine median, ulnar, and radial sensory nerve conduction studies are normal. EMG reveals decreased recruitment and 4+ positive sharp waves in infraspinatus, brachialis, deltoid, brachioradialis, pronator teres, and triceps, but normal activity in extensor indicis proprius, flexor digitorum superficialis, abductor pollicis brevis, and first dorsal interosseous. Which of the following is the most likely diagnosis?

A

C6 radiculopathy
-C6 radiculopathy is the best answer. All muscles with some C6 innervation are affected here, showing active denervation (positive sharp waves). Indeed, many of these muscles have C5 innervation and are affected, but this continues in muscles without C5 innervation (pronator teres: C6-C7, triceps: C6-C7-C8); notably, muscles with C7 or C8 or T1 innervation (lacking any C6) are spared. Sensory nerve action potentials are also normal, indicating proximal (e.g. radicular) process rather than distal. Posterior cordopathy would spare pronator teres. Upper trunk plexopathy would show median sensory abnormalities on NCS, as well as lateral antebrachial cutaneous nerve SNAP abnormality (which was untested here). Please refer to the innervation chart at the beginning of the Upper and Lower Extremity Peripheral Nervous System Diseases chapters for a detailed innervation guide.

How well did you know this?
1
Not at all
2
3
4
5
Perfectly
52
Q

During an EMG, you ask the patient to very lightly begin contracting their extensor indicis proprius. Instantly you notice the EMG screen become flooded with many small, short-duration MUAPs. Which of the following is the most likely diagnosis?

A

Polymyositis
- This question describes a myopathic recruitment pattern, or increased recruitment pattern. This is typically seen in myopathies, not neuropathies; thus, polymyositis is correct.

How well did you know this?
1
Not at all
2
3
4
5
Perfectly
53
Q

You are reviewing left lower limb EMG/NCS findings and trying to propose a possible treatment plan to the patient. The patient is experiencing foot pain not relieved by NSAIDs or heat/ice. NCS reveals normal superficial fibular and sural SNAPs (sensory nerve action potentials), prolonged latency of medial and lateral plantar nerve SNAPs, and abnormal CMAP (compound muscle action potential) to abductor hallucis (AH). The CMAP to the extensor digitorum brevis (EDB) is normal. Needle EMG reveals decreased recruitment in AH, abductor digiti quinti pedis (ADQP), and lumbricals. EMG of tibialis anterior, EDB, gastrocnemius, fibularis longus, hamstrings, rectus femoris, tensor fascia lata, and lumbosacral paraspinals is normal. Which of the following is the most reasonable treatment strategy for this condition?

A

Bracing
-These findings suggest tarsal tunnel syndrome, a rare compression of the tibial nerve as it passes through the tarsal tunnel around the medial malleolus, underneath the flexor retinaculum. The flexor retinaculum is a common culprit that compresses the tibial nerve too tightly, causing the patient’s symptoms of plantar foot pain with numbness/tingling and possible foot muscle weakness. Surgical release of the flexor retinaculum (akin to carpal tunnel release) is the best option of these choices (a structural solution for a structural problem). US-guided tarsal tunnel injection could also be considered. Bracing would not do anything to decompress the tarsal tunnel. Gabapentin also would not decompress the tarsal tunnel, but mask the symptoms while the nerve compression continues, potentially destroying axons over time. Peripheral nerve stimulation is not appropriate for this reason also.

How well did you know this?
1
Not at all
2
3
4
5
Perfectly
54
Q

During EMG studies, decreased insertional activity can be seen in which of the following situations?

A

Muscle fibrosis can cause decreased insertional activity. Increased insertional activity can be seen in cases in which the muscle cell membrane is hyper-irritable, such as active denervation

How well did you know this?
1
Not at all
2
3
4
5
Perfectly
55
Q

A 58 year-old female complains of sudden-onset severe shoulder pain for 2 weeks, which then improves and is followed by progressive onset shoulder weakness. She denies trauma. Shoulder x-ray and MRI are negative. On exam she has weakness in shoulder abduction. EMG/NCS reveals decreased recruitment and +2 positive sharp waves in the supraspinatus and infraspinatus. Which of the following is the most reasonable advice you should give to this patient?

A

This patient presents with classic Parsonage-Turner Syndrome (neuralgic amyotrophy, idiopathic brachial plexopathy). This typically presents as sudden shoulder pain for 2 weeks which gives way to weakness, classically in the suprascapular (this case), long thoracic, and/or anterior interosseous nerve territory. Serial EMGs are useful for prognosis, and most cases self-resolve within 1-2 years after onset. Early surgical options would not be indicated, as most cases self-resolve. PTS typically begins following a viral illness or surgery, and its etiology is not entirely defined at this time.

How well did you know this?
1
Not at all
2
3
4
5
Perfectly
56
Q

A patient presents with deep fibular neuropathy. Which of the following areas of the foot do you expect to demonstrate sensory abnormalities?

A

1st webspace, sparing the dorsum of the foot
-The deep fibular nerve innervates the skin between the 1st and 2nd toes (the 1st webspace). The superficial fibular nerve innervates the skin over the entire dorsum of the foot, except the 1st webspace (which is deep fibular nerve territory).

How well did you know this?
1
Not at all
2
3
4
5
Perfectly
57
Q

With LCL (lateral collateral ligament) injuries in the knee, it is important to rule out nerve injury most crucially in which of the following nerves?

A

The common fibular nerve wraps around the fibular head/neck near the LCL attachment, so with injuries to the LCL come injuries to other nearby structures, e.g. the common fibular nerve; thus, it is important to rule out injury to this nerve in a patient with LCL tear. High-yield functions to test would be foot dorsiflexion and eversion.

How well did you know this?
1
Not at all
2
3
4
5
Perfectly
58
Q

A 29 year-old female presents to your clinic with complaints of 2 months of bilateral hand numbness and tingling, worse at night. She is expecting her first child later this year. On exam, strength and sensation are intact. What is the next best step?

A

Bracing
-Pregnancy can cause carpal tunnel syndrome, simply due to fluid balance changes in the body. The first step is a carpal tunnel wrist brace that is to be worn during sleep, which prevents wrist flexion (wrist flexion causes increased pressure within the carpal tunnel, compressing and irritating the median nerve over time)

How well did you know this?
1
Not at all
2
3
4
5
Perfectly
59
Q

A 25 year-old male develops 2 weeks of progressive lower limb weakness. He has a history of recent gastroenteritis. Bilateral lower limb EMG/NCS reveals prolonged latency of bilateral superficial fibular nerve SNAPs (sensory nerve action potentials), but normal sural nerve SNAPs. The bilateral CMAPs to the EDB and AH (compound muscle action potentials to the extensor digitorum brevis and abductor hallucis) show prolonged latency but normal amplitude. EMG reveals decreased recruitment in the bilateral EDB, AH, tibialis anterior, gastrocnemius, and semimembranosus, but normal activity in rectus femoris and adductor longus. Which of the following additional EMG/NCS findings would most likely be discovered in this patient?

A

This patient presents with Guillain-Barre Syndrome (GBS), also referred to as AIDP (acute inflammatory demyelinating polyradiculopathy). GBS is caused by a recent infection (typically GI or URI) causing your immune system to confuse a foreign protein with a protein on your own myelin. Thus, your body destroys your own peripheral nerve myelin (demyelinating the nerves and prolonging the latency). The first EDX finding of GBS is delayed or absent F-waves. NCS reveals prolonged latency and sometimes reduced amplitude of SNAPs with sural nerve sparing, as the sural nerve is larger with more myelin than other peripheral sensory nerves, and thus is less affected than the other nerves. CMAPs show prolonged latency, decreased conduction velocity (CV), and typically normal amplitude (this is a demyelinating disease primarily, not axonal loss). EMG shows decreased recruitment but otherwise there should be no active denervation usually, but this can occur as a secondary feature of GBS weeks after onset. Treatment involves PT and early plasmapheresis/IVIG. Abnormal/increased temporal dispersion is also a feature of this disease on NCS, due to the widespread and more uniform demyelination of nerves taking place.

How well did you know this?
1
Not at all
2
3
4
5
Perfectly
60
Q

During a nerve conduction study, you decide to place the recording electrode over the patient’s soleus, and the reference electrode over their achilles tendon. You then stimulate in a proximal direction from the popliteal fossa. When you examine the waveform produced by this, you note prolonged latency. What is the most appropriate conclusion from these results?

A

There is damage somewhere along the pathway you stimulated
-The acquisition of the H reflex is described here. By performing this, the physician stimulates the Ia sensory afferent nerves and records over the muscle (soleus), sending the action potential proximally toward the spinal cord, triggering the spinal reflex arc, and sending the action potential back down distally along the motor nerve (sciatic → tibial) to make the muscle (soleus) contract. Prolonged latency of this clearly very long nerve pathway merely indicates that there is some kind of damage somewhere along the pathway that has caused the signal to take much longer to reach the recording electrode than is normal. The H reflex study is typically used to assess for S1 radiculopathy.

How well did you know this?
1
Not at all
2
3
4
5
Perfectly
61
Q

A 25 year-old female is involved in a motor vehicle accident. She suffers a left proximal femur fracture and requires orthopedic surgery to repair her left hip. She presents to your inpatient rehabilitation unit 2 weeks later for functional restoration. On exam, you note intact right lower limb strength, intact left hip flexion, knee extension, adduction, internal/external rotation, and abduction strength, but impaired knee flexion, dorsiflexion, and inversion. She has paresthesias in her posterior thigh and anterior shin. What is the most likely diagnosis?

A

This patient presents with findings concerning for sciatic neuropathy, either as a result of the trauma itself, or the surgical approach (posterior approach places sciatic nerve at risk). Proximal sciatic neuropathy will affect essentially all sciatic-innervated muscles, including hamstrings, and all muscles originating at or below the knee. Thus, knee flexion is impaired (hamstrings), dorsiflexion is impaired (common fibular nerve, which arises from sciatic nerve), and inversion is impaired (tibial nerve, which arises from sciatic nerve). Plantarflexion would also be impaired (gastroc/soleus, tibial nerve), but this was not mentioned in the exam. Femoral neuropathy would cause impaired knee extension (intact in this patient); Lumbosacral plexopathy would cause patchy abnormalities not likely cleanly narrowed down to a single nerve (as in this case); tibial and common fibular neuropathies would indeed cause the abnormalities discussed here, except the impaired knee flexion and paresthesias in the posterior thigh imply a proximal sciatic nerve pathology, leading to weak hamstrings.

How well did you know this?
1
Not at all
2
3
4
5
Perfectly
62
Q

During an upper limb EMG/NCS study you note normal sensory nerve action potentials (SNAPs) to digits 1, 2, and 5. Compound motor action potentials (CMAPs) to the abductor pollicis brevis (APB) and first dorsal interosseous (FDI) are normal. Needle EMG reveals decreased recruitment and abnormal spontaneous activity in the brachialis and biceps, with normal EMG of cervical paraspinals, triceps, brachioradialis, pronator teres, APB, and FDI. Which of the following would also most likely be found in this study?

A

This patient presents with findings consistent with musculocutaneous neuropathy. This nerve innervates the coracobrachialis (not discussed here), biceps brachii, and brachialis, hence the abnormal EMG to the biceps and brachialis. The musculocutaneous nerve terminates as the LAC; thus, LAC SNAP should be studied and would be expected to be abnormal compared to the contralateral side. Absent CMAP to EIP could be seen in posterior interosseous neuropathy or a proximal radial neuropathy. Radial neuropathy above the humerus midshaft could cause abnormal EMG to the anconeus. Abnormal CMAP to the FCR could be caused by median neuropathy, but not musculocutaneous neuropathy.

How well did you know this?
1
Not at all
2
3
4
5
Perfectly
63
Q

Which of the following is the key NCS/EMG finding, found in radiculopathies, that helps to differentiate a radiculopathy from more peripheral nerve lesions?

A

Normal SNAPs (sensory nerve action potentials) are the key NCS finding demonstrated in a radiculopathy. Because the dorsal root ganglion contains the cell bodies of the sensory neurons of a dermatome of a given limb, and these cells are bipolar neurons residing outside the spinal cord, any nerve injury proximal to these cell bodies (e.g. radiculopathy) will not affect the distally directed axons of these bipolar cells (i.e. the projections extending down the arm or leg from the dorsal root ganglion). Thus, these unaffected axons will conduct impulses quite normally, and SNAPs will be entirely normal in a pure radiculopathy. Paraspinal muscles corresponding to the affected nerve root level will theoretically show EMG changes (increased irritability, possible active denervation). CMAPs will be expected to potentially be abnormal, but this would not be the key differentiating feature of a radiculopathy.

How well did you know this?
1
Not at all
2
3
4
5
Perfectly
64
Q

In a nerve conduction study, the _______ is _______ charged, thus attracting _______ towards it.

A

On the stimulating electrode during a nerve conduction study, the cathode is negatively charged, which attracts positive sodium ions towards it; these sodium ions accumulate right outside the axon membrane where their concentration becomes so large that they trigger the voltage-gated sodium channels to open, thus initiating an action potential.

How well did you know this?
1
Not at all
2
3
4
5
Perfectly
65
Q

Which of the following muscles receives its innervation via the following neural pathway? C7, C8, T1; middle and lower trunk; medial and lateral cord; median nerve; anterior interosseous nerve.

A

PQ is innervated by the above pathway. FDS, FCR, and lumbricals 1 and 2 are all directly innervated by the median nerve, not the anterior interosseous nerve.

How well did you know this?
1
Not at all
2
3
4
5
Perfectly
66
Q

Normal motor recruitment follows which of the following principles?

A

Small motor units are recruited before large units. Type II motor units are fast twitch, powerful fibers that are recruited with maximal intensity, thus filling the EMG screen with motor units by the time Type II units are recruited.

How well did you know this?
1
Not at all
2
3
4
5
Perfectly
67
Q

The suprascapular nerve innervates the supraspinatus. It then passes through which structure on its way to innervate the infraspinatus?

A

The suprascapular nerve passes through the spinoglenoid notch to innervate the infraspinatus.

68
Q

A trauma results in complete severing of the median nerve at the elbow. Immediate EMG/NCS reveals absent CMAP proximally, and normal CMAP distally. Which of the following is the most likely diagnosis?

A

Neurotmesis is the severing (transection) of a nerve all the way through the epineurium, due to trauma. Initially we see an absent CMAP proximally and normal CMAP distally. Once Wallerian degeneration is complete (roughly 5 days for motor fibers), we see absent CMAP both distally and proximally to the site of injury.

69
Q

During an EMG, you decide to stimulate the ulnar nerve at the elbow and record the CMAP (compound muscle action potential) over the abductor digiti minimi (ADM). You then stimulate the ulnar nerve at the wrist and notice a much higher amplitude at the wrist than when stimulating at the elbow. Which of the following is the most likely etiology of these findings?

A

Martin-Gruber anastomosis
-The Martin-Gruber anastomosis involves median nerve fibers in the forearm crossing over and joining the ulnar nerve in the forearm. This means essentially that the median nerve innervates the abductor digiti minimi (ADM) and first dorsal interosseous (FDI), which are ulnar nerve muscles, in addition to innervating its own classic median nerve muscles (abductor pollicis brevis - APB, etc.). The ulnar nerve CMAP (compound muscle action potential) when stimulating at the elbow will show a low amplitude when recording over a distal ulnar muscle such as FDI. The ulnar CMAP will be “repaired”/normal if you stimulate the ulnar nerve at the wrist. Thus, it appears that there is a conduction block somewhere in the forearm when there actually is not (ulnar nerve conduction block in the mid to distal forearm is also very uncommon). When you stimulate the ulnar nerve at the wrist, at this point the median nerve fibers have finally joined the ulnar nerve, so you are finally stimulating all the motor axons that are supplying the ulnar nerve hand muscles. When you stimulate the ulnar nerve at the elbow, you are not stimulating the median nerve fibers that have yet to contribute to the ulnar innervations, thus you only generate part of the full amplitude, which is what makes it look like there is a conduction block in the ulnar nerve, when in reality there is not.

70
Q

Which of the following muscles is innervated by the listed neural pathway? S1, S2; sciatic nerve; tibial nerve; medial plantar nerve.

A

The FHB is innervated by this pathway, as well as the abductor hallucis brevis (AHB). The FHL and FDL are tibial nerve-innervated. The ADQP is lateral plantar nerve-innervated.

71
Q

An axon is regrowing down a nerve. Finally, after many months of searching, it has returned to its original muscle fibers that it used to innervate. However, to its dismay, it discovers that another axon has sprouted collateral branches which are currently supplying our hero axon’s former muscle fibers. The two axons decide to compete for muscle fiber dominance. Which axon will ultimately prevail and maintain control of these muscle fibers?

A

Whichever axon possesses the largest size with the strongest neuromuscular junction connections
-The biggest motor neuron with the strongest NMJ connection will “win” control of a given muscle fiber.

72
Q

Which of the following EMG/NCS findings is associated with post-polio syndrome? MUAP: motor unit action potential. SNAP: sensory nerve action potential. CMAP: compound muscle action potential.

A

Giant MUAPs
-Post-polio syndrome (“burning out” and death of anterior horn cells in a patient with a history of polio who recovered decades ago) is associated with giant MUAPs on EMG. This is essentially because a few anterior horn cells are doing all the work of the contracting the muscles, and so there are very few MUAPs to study, as all the muscle fibers belong to just a few anterior horn cells. SNAPs are unaffected in anterior horn cell disease. CMAP amplitudes are low in post-polio syndrome. CMAP decrement during repetitive nerve stimulation is indicative of neuromuscular junction disease, not post-polio syndrome.

73
Q

Which of the following muscles is NOT innervated by the anterior interosseous nerve?

A

Flexor digitorum superficialis
-FDS is innervated directly by the median nerve, not the anterior interosseous nerve (which arises from the median nerve).

74
Q

During a blink reflex study, when stimulating the right trigeminal nerve, you notice both eyes blink. This is known as which of the following?

A

R2 response
- The bilateral blink response observed during a blink reflex study is the R2 response. The quick, ipsilateral blink response is called the R1 response, and we record it from the orbicularis oculi muscle ipsilaterally to the side of stimulation. Synkinesis is the result of inappropriate regeneration of the facial nerve, causing facial nerve function abnormalities.

75
Q

Which of the following muscles is not innervated by the posterior cord of the brachial plexus?

A

The rhomboids’ innervation arises from C4, C5 dorsal scapular nerve (pre-plexus!). The remaining muscles are all posterior cord-innervated muscles.

76
Q

A patient’s right lower limb EMG/NCS results are as follows. Routine fibular and tibial motor/sensory nerve conduction studies are normal. H-reflex shows prolonged latency compared to the contralateral side. EMG reveals decreased recruitment and 1+ fibrillations in tensor fascia lata, gluteus maximus, extensor hallucis longus, long head of biceps femoris, fibularis longus, gastrocnemius, and abductor hallucis, but normal activity in rectus femoris, adductor longus, tibialis anterior, and extensor digitorum longus. Which of the following is the most likely diagnosis?

A

All S1 muscles being affected, combined with abnormal H-reflex (indicates some sort of lesion along the long S1 pathway, typically used to help diagnose an S1 radiculopathy), and normal activity in all muscles lacking any S1 innervation, indicates S1 radiculopathy as the best answer.

77
Q

In a patient with spinal cord injury, what information or test may help you distinguish between cauda equina syndrome and conus medullaris syndrome?

A

EMG
-EMG assesses lower motor neurons and their muscle fibers exclusively. It does not tell us anything about upper motor neurons or the spinal cord. Thus, if the EMG is abnormal, it points to a cauda equina injury, as the cauda equina is purely lower motor neuron fibers that have already sprouted off the ventral horn of the spinal cord. If the EMG is normal, it points to a purely upper motor neuron injury, i.e. conus medullaris lesion instead of cauda equina; if the cauda equina were involved you would see abnormalities on EMG for lower limb muscles. It is true that the conus medullaris is located at L1, but simply knowing the trauma level of L1 does not tell us enough information about exactly what nervous structures were injured, as the cauda equina fibers are also located at that spot.

78
Q

A 34 year-old male presents to your clinic for the evaluation of left periscapular pain. He has a history of motorcycle accident 1 year ago and has had this pain ever since then. On exam, you note his left scapula is positioned further laterally than the right scapula. Which nerve is implicated in this condition?

A

Long thoracic nerve
-With a laterally winged scapula, the spinal accessory nerve commanding the trapezius is deficient. With a medially winged scapula, the long thoracic nerve commanding the serratus anterior would be deficient. The thoracodorsal nerve innervates the latissimus dorsi and does not cause a winged scapula when injured. The upper subscapular nerve innervates the subscapularis and does not cause a winged scapula when injured.

79
Q

Regarding the dorsal and palmar interossei of the hand, which of the following statements is true?

A

They share the same innervation
-The dorsal and palmar interossei of the hand are all innervated by the ulnar nerve. The first dorsal interosseous (FDI) is routinely needled in an EMG study as a valuable, accessible ulnar nerve-innervated muscle in the hand.

80
Q

You are reviewing EMG/NCS results of a patient. The results demonstrate decreased proximal and distal amplitude with normal latency. EMG reveals decreased recruitment. Which of the following is the most likely etiology of these findings?

A

Axonal loss-
Decreased amplitude throughout an entire nerve’s length suggests axonal loss. EMG will show decreased recruitment in these cases. Conduction block is seen with proximally reduced amplitude, but stimulating distally beyond the spot of conduction block can demonstrate a normal “repaired” amplitude.

81
Q

A patient’s left lower limb EMG/NCS results are as follows. Routine fibular and tibial motor/sensory nerve conduction studies are normal. EMG reveals decreased recruitment and polyphasic potentials in rectus femoris and adductor longus, but normal activity in tensor fascia lata, semimembranosus, tibialis anterior, gastrocnemius, extensor digitorum brevis, and abductor hallucis. Which of the following is the most likely diagnosis?

A

L3 radiculopathy
-L3 radiculopathy is correct. Two different L3-innervated muscles with different peripheral nerve innervation are affected in the setting of normal sensory nerve action potentials, localizing this as a radiculopathy to the L3 nerve root. L4 radiculopathy would affect the tibialis anterior, extensor digitorum brevis, and potentially the hamstrings and tensor fascia lata. Femoral neuropathy would spare the adductor longus, and obturator neuropathy would spare the rectus femoris. Please refer to the innervation chart at the beginning of the Upper and Lower Extremity Peripheral Nervous System Diseases chapters for a detailed innervation guide.

82
Q

Which of the following muscles does not share L4 root innervation?
Gluteus maximus, Gluteus medius, Tibialis anterior, Iliopsoas

A

Innervations for these muscles are as follows. Iliopsoas: L2, L3, L4. Tibialis anterior: L4, L5. Gluteus medius: L4, L5, S1. Gluteus maximus: L5, S1, S2.

83
Q

A 66 year-old male develops right leg pain and weakness following a hospital stay for acute coronary syndrome requiring cardiac stenting. On exam he demonstrates isolated knee extension weakness. On EMG/NCS he demonstrates abnormal CMAP (compound muscle action potential) to the rectus femoris. Needle EMG reveals decreased recruitment and 1+ fibrillations in the vastus medialis. In order to cinch the diagnosis, you decide to perform additional studies, and expect to find which of the following findings? SNAP: sensory nerve action potential.

A

Decreased amplitude of saphenous SNAP
- patient presents with femoral neuropathy due to cardiac catheter trauma (inserting the catheter into the femoral artery - next to the femoral nerve). The femoral nerve terminates as the saphenous nerve, supplying skin innervation to the medial leg below the knee. Thus, the saphenous nerve SNAP being abnormal (decreased amplitude would be expected) would nicely complete the diagnosis of femoral neuropathy in this patient.

84
Q

You ask your patient to abduct and externally rotate their shoulders to 90 degrees and then begin opening and closing their hands over a 3-minute period. This test is known by which name, and evaluates which condition?

A

Roos, lower trunk brachial plexopathy
- This question describes the Roos test, which assesses neurogenic thoracic outlet syndrome, which itself is a lower trunk plexopathy. A positive test is indicated by reproduction of the patient’s symptoms in the upper limb (pain, numbness, tingling in a C8-T1 pattern).

85
Q

A 30 year-old male is involved in a motorcycle collision and develops sudden-onset severe right arm pain. He has bruising and swelling of his upper arm on exam. Xrays reveal a mid-shaft humeral fracture. Which of the following muscles is most important to test for strength?

A

Extensor carpi radialis brevis (ECRB)
- Midshaft humeral fractures commonly cause radial neuropathy due to the close proximity of the radial nerve to the humerus at this location. At the level of the midshaft of the humerus, the triceps and anconeus are already innervated. Thus, you must examine downstream radial nerve muscles to assess integrity of the radial nerve. In this case, ECRB is the only muscle listed that has yet to be innervated by the radial nerve; thus, it should be affected if the radial nerve has been damaged by this fracture. A positive exam would yield impaired strength in wrist extension.

86
Q

Which of the following nerves is associated with the spinoglenoid notch?

A

Suprascapular nerve
-The suprascapular nerve innervates the supraspinatus, then passes through the spinoglenoid notch to innervate the infraspinatus. This is an important anatomical consideration when performing an EMG involving both supraspinatus and infraspinatus. The axillary nerve passes through the quadrilateral space of the axilla.

87
Q

During a blink reflex study, you recall that the Vm nucleus is located within which of the following structures?

A

Pons
-The blink reflex study tests CN V and CN VII in the brainstem and peripherally. CN V has two nuclei being tested (Vm and Vs). The Vm nucleus lies within the pons. It accepts CN V input from the face and conducts it to the ipsilateral CN VII nucleus, causing an ipsilateral blink (R1 response). It also simultaneously conducts the impulse to the medulla where the Vs nucleus lies. Vs accepts the impulse from Vm and sends the impulse to the bilateral CN VII nuclei, which then causes a bilateral blink via the orbicularis oculi muscles (R2 response).

88
Q

Which of the following involves the postsynaptic portion of the neuromuscular junction?

A

Myasthenia Gravis (MG)
- LEMS and botulism are presynaptic neuromuscular junction (NMJ) disorders. Post-Polio Syndrome involves the burning out and subsequent death of anterior horn cells that survived a patient’s remote history of polio. MG is truly a postsynaptic NMJ disease involving antibodies against the acetylcholine receptor on the postsynaptic membrane.

89
Q

A Martin-Gruber anastomosis is an anatomic variant consisting of which of the following nerve findings?

A

Median to ulnar anastomosis
- The Martin-Gruber anastomosis involves median nerve fibers in the forearm crossing over and joining the ulnar nerve in the forearm. This means essentially that the median nerve innervates the abductor digiti minimi (ADM) and first dorsal interosseous (FDI), which are ulnar nerve muscles, in addition to innervating its own classic median nerve muscles (abductor pollicis brevis - APB, etc.). The ulnar nerve CMAP (compound muscle action potential) when stimulating at the elbow will show a low amplitude when recording over a distal ulnar muscle such as FDI. The ulnar CMAP will be “repaired”/normal if you stimulate the ulnar nerve at the wrist. Thus, it appears that there is a conduction block somewhere in the forearm when there actually is not.

90
Q

During an EMG study, needling of the flexor pollicis longus and pronator quadratus reveal 2+ fibrillations and positive sharp waves, while the remaining muscles (abductor pollicis brevis, first dorsal interosseous, flexor carpi radialis, biceps brachii, triceps, cervical paraspinals) are normal. Which of the following is the most likely diagnosis?

A

Anterior interosseous neuropathy
- The flexor pollicis longus, pronator quadratus, and flexor digitorum profundus to digits 2,3 are all innervated by the anterior interosseous nerve (AIN). Thus, anterior interosseous neuropathy is correct. Posterior interosseous neuropathy (PIN-opathy) would show abnormal EMG to PIN-innervated muscles (extensor digitorum, extensor indicis proprius, extensor carpi ulnaris, extensor pollicis longus). Median neuropathy at the elbow would show abnormalities in median nerve muscles (flexor carpi radialis, for example) in addition to some AIN muscles. Medial cord-opathy would show changes in other medial cord muscles, such as the first dorsal interosseous, which was normal in this study.

91
Q

When listening to a muscle at rest, you see a signal that appears like a normal motor unit action potential (MUAP) and occurs at irregular intervals. What is the proper term for this signal?

A

Fasciculation
- When This question describes a fasciculation, which can be seen in normal patients, and classically in ALS patients as well. ALS = amyotrophic lateral sclerosis.

92
Q

A patient with weakness presents for repetitive nerve stimulation (RNS). You decide to evaluate the compound muscle action potential (CMAP) of the abductor digiti minimi (ADM). On routine studies, you notice an abnormally low amplitude of the CMAP. During RNS while stimulating the muscle at a rate of 2 Hz, you notice a 20% decrement in the CMAP amplitude when comparing the 1st and 4th waveforms. However, during RNS while stimulating the muscle at a rate of 50 Hz, you notice a surge in the CMAP amplitude, increasing its size by 300%. Which of the following diseases do you suspect?

A

Lambert-Eaton Myasthenic Syndrome (LEMS)
- The clinical scenario describes a patient with muscle weakness and specific findings on repetitive nerve stimulation (RNS). The key findings are a 20% decrement in CMAP amplitude at low-frequency stimulation (2 Hz) and a 300% increase in CMAP amplitude at high-frequency stimulation (50 Hz). These findings are characteristic of Lambert-Eaton Myasthenic Syndrome (LEMS). In LEMS, there is a presynaptic defect in neuromuscular transmission due to antibodies against voltage-gated calcium channels, leading to reduced release of acetylcholine. Low-frequency stimulation shows a decremental response, while high-frequency stimulation or exercise leads to an incremental response due to increased calcium influx and subsequent acetylcholine release. This pattern is not typical for ALS, MG, or Botulism.

93
Q

Which of the following muscles is innervated by the lateral cord of the brachial plexus?

A

Flexor carpi radialis
- The deltoid and brachioradialis are innervated by the posterior cord. The FDP 2,3 is innervated by the medial cord. The FCR is innervated by the lateral cord.

94
Q

Motor nerve conduction studies are typically recorded in which way?

A

Orthodromically
- Motor NCS are recorded orthodromically; the nerve is shocked, and the action potential travels distally down the axon just like it does in everyday life. Sensory nerve studies are typically recorded antidromically.

95
Q

A patient’s left upper limb EMG/NCS results are as follows. Routine median, ulnar, and radial sensory nerve conduction studies are normal, except for decreased amplitude of the sensory nerve action potential (SNAP) to the anatomic snuffbox. EMG reveals decreased recruitment and 1+ fibrillations in deltoid, brachioradialis, and triceps, but normal EMG in biceps, pronator teres, flexor pollicis longus, abductor pollicis brevis, and first dorsal interosseous. Which of the following muscles would also be expected to exhibit EMG abnormalities in this patient?

A

Extensor indicis proprius
-This patient suffers from a posterior cord plexopathy, as evidenced by all posterior cord-innervated muscles being affected, and all non-posterior-cord muscles being spared, in addition to the radial nerve SNAP (snuffbox) being abnormal. Of the answer choices, only EIP is a posterior cord muscle; thus, EIP is correct.

96
Q

During an upper limb EMG/NCS study, you detect decreased SNAP amplitude to digit 5 and to the dorsal ulnar cutaneous nerve. EMG reveals decreased recruitment to the abductor digiti minimi and flexor digitorum profundus to digits 4 and 5. Which of the following is most likely to narrow down the diagnosis?

A

Testing the medial antebrachial cutaneous nerve SNAP
- These EMG/NCS results suggest either ulnar neuropathy at the elbow or medial cord brachial plexopathy. The medial antebrachial cutaneous nerve (MAC) branches off the medial cord of the brachial plexus to supply cutaneous innervation to the medial forearm just before the medial cord continues down to form the ulnar nerve. Thus, an abnormal MAC would indicate medial cord plexopathy (or lower trunk plexopathy), indicating that the nerve problem is somewhere proximal to the ulnar nerve, NOT at the ulnar nerve. However, a normal MAC would indicate that the medial cord of the plexus is intact, thus localizing the problem distally (likely the ulnar nerve at the elbow in this case). Comparing the DUC between both sides is not helpful, as we already know that ours being studied is abnormal. Obtaining the lateral antebrachial cutaneous SNAP (arises as a continuation of the musculocutaneous nerve from the lateral cord of the plexus) will not inform us on the status of the medial cord of the plexus, thus making this less helpful to us in this case. Finally, needling the FCU may show abnormalities in this muscle, which could be caused by medial cord plexopathy or ulnar neuropathy at the elbow; thus, this would not help us narrow our diagnosis.

97
Q

A 28 year-old male presents for a right upper limb EMG/NCS. During the study you detect decreased amplitude of the sensory nerve action potential (SNAP) to the anatomic snuffbox, with normal SNAPs to digits 2 and 5. Compound motor action potentials (CMAPs) to the extensor indicis proprius (EIP), abductor pollicis brevis (APB), and first dorsal interosseous (FDI) are normal. Needle EMG of the cervical paraspinals, deltoid, biceps brachii, triceps, brachioradialis, EIP, APB, and FDI is normal. Which of the following is the most likely cause of the patient’s symptoms?

A

Tight wristwatch
-patient presents with Cheiralgia Paresthetica - essentially superficial radial neuropathy, a pure sensory syndrome, hence the abnormal radial SNAP (snuffbox SNAP) but normal CMAPs and needle EMG. A tight wristwatch, or handcuffs, or procedural trauma, could cause injury to the superficial radial nerve, thus causing pain over the radial nerve territory of the hand. This can be detected by abnormal SNAPs to the anatomic snuffbox and radial SNAP to digit 1 (the thumb). Midshaft humerus fractures classically produce radial neuropathy that spares the triceps and anconeus (as these muscles are innervated prior to the humeral midshaft area - the spiral groove) but shows abnormalities in other downstream radial nerve-innervated muscles. Improper crutch use can cause a proximal radial neuropathy, but that would demonstrate EMG abnormalities in the triceps/anconeus, and the triceps is normal in this patient’s study. Compression by the Arcade of Frohse can cause posterior interosseous neuropathy (PIN-opathy) - a pure motor neuropathy that would spare brachioradialis (not a PIN-innervated muscle) and show normal radial SNAPs (our radial SNAP in this patient’s study is abnormal).

98
Q

You are reviewing EMG/NCS findings. You note abnormal superficial fibular and sural SNAPs (sensory nerve action potentials), normal medial and lateral plantar nerve SNAPs, and abnormal CMAP to the EDB (compound muscle action potential to extensor digitorum brevis). The needle EMG results demonstrate decreased recruitment in tibialis anterior (TA), EDB, and fibularis longus, with normal activity in abductor hallucis, gastrocnemius, semimembranosus, biceps femoris, rectus femoris, tensor fascia lata, and lumbar paraspinals. Which of the following is the most likely diagnosis?

A

Common fibular neuropathy
- Common fibular neuropathy is the best answer. Deep fibular neuropathy would spare the fibularis longus (abnormal in this case). Superficial fibular neuropathy would spare the EDB and TA (abnormal in this case). Sciatic neuropathy would show abnormalities in gastrocnemius, abductor hallucis, and plantar SNAPs potentially, as well as potentially hamstring muscles.

99
Q

Which of the following EMG/NCS findings would indicate nerve compression by the Arcade of Frohse? SNAP: sensory nerve action potential. BR: brachioradialis. EIP: extensor indicis proprius.

A

Normal SNAPs to digits 1,2,5, snuffbox. Normal EMG to BR. Abnormal EMG to EIP
- Compression by the Arcade of Frohse can cause posterior interosseous neuropathy (PIN-opathy) - a pure motor neuropathy that would spare brachioradialis (not a PIN-innervated muscle) and show normal radial SNAPs. “Normal SNAPs to digits 1,2,5, snuffbox. Normal EMG to BR. Abnormal EMG to EIP” is the most correct answer choice. “Normal SNAPs to digits 1,2,5; abnormal snuffbox SNAP. Abnormal EMG to BR and EIP” and “Normal SNAPs to digits 1,2,5, abnormal snuffbox SNAP. Normal EMG to BR. Abnormal EMG to EIP” show abnormal radial SNAP, thus they are incorrect. Choice “Normal SNAPs to digits 1,2,5, snuffbox. Abnormal EMG to BR and EIP” shows abnormal BR EMG, which would be spared in a PIN-opathy.

100
Q

Which of the following genetic sequences is associated with Charcot-Marie-Tooth (CMT) Neuropathy type Ia?

A

CMT1A is associated with duplication of the PMP-22 gene. Hereditary neuropathy with liability to pressure palsy (HNPP) is associated with deletion of PMP-22. Mutations of the SMN-1 gene are the cause of spinal muscular atrophy and its subtypes.

101
Q

During an upper limb EMG/NCS you detect normal sensory nerve action potentials (SNAPs) to digits 1,2, and 5. Compound muscle action potentials (CMAPs) to the abductor pollicis brevis (APB) and first dorsal interosseous (FDI) are normal. Needle EMG to the deltoid and teres minor shows decreased recruitment and 3+ positive sharp waves, while the biceps brachii, triceps, pronator teres, FDI, and APB are normal. Which of the following is most likely associated with this condition?

A

Improper use of crutches
-The needle EMG results localize this injury to the axillary nerve (axillary neuropathy), which innervates the deltoid and teres minor. These muscles demonstrate active denervation in this study, indicative of axonal loss. A posterior cord plexopathy would produce abnormal EMG in the triceps, so the lesion is more distal, i.e. in the axillary nerve. A CMAP to the deltoid would be abnormal in our patient as well, but that is not mentioned in our patient’s study. Winged scapula is associated with long thoracic neuropathy and spinal accessory neuropathy. Spiral groove fractures cause radial neuropathy. Myokymia is seen in upper trunk brachial plexopathy. Improper crutch use can cause axillary neuropathy; thus, this is the correct answer.

102
Q

Once a patient has reached age 50, the conduction velocity of their nerves will decrease by approximately how many meters per second for each subsequent decade?

A

2
- After age 50, conduction velocity normally decreases by about 2 m/s per decade

103
Q

A patient’s right upper limb EMG/NCS results are as follows. Routine median, ulnar, and radial sensory nerve conduction studies are normal except for reduced amplitude of the median and radial SNAPs to the thumb. EMG reveals decreased recruitment and 1+ positive sharp waves in supraspinatus, deltoid, biceps, triceps, brachioradialis, and flexor carpi radialis, but normal activity in rhomboids, extensor indicis proprius, flexor digitorum superficialis, abductor pollicis brevis, and first dorsal interosseous. Which of the following is the most likely diagnosis?

A

Upper trunk brachial plexopathy
- All upper trunk muscles and sensory nerves noted are affected (median SNAP to the thumb is from C6/upper trunk fibers!). Radiculopathy will show normal sensory nerve action potentials (SNAPs). Lateral cord plexopathy would affect biceps, brachialis, coracobrachialis, pronator teres, flexor carpi radialis, flexor digitorum superficialis, pronator quadratus, flexor pollicis longus, pectoralis major, and lateral antebrachial cutaneous nerve. Upper trunk plexopathy is the best answer. Please refer to the innervation chart at the beginning of the Upper and Lower Extremity Peripheral Nervous System Diseases chapters for a detailed innervation guide.

104
Q

During nerve conduction studies, raising the low frequency filter will cause which of the following effects?

A

Decreased amplitude
- Raising the low frequency filter will decrease the amplitude, as will lowering the high frequency filter.

105
Q

The first sign of Guillain Barre Syndrome on electrodiagnostic studies is which of the following?

A

Prolonged F wave
- Prolonged or absent F waves are the first sign of GBS on EDX studies.

106
Q

The rhomboids are innervated by which trunk of the brachial plexus?

A

None of these answers is correct
- The rhomboids are innervated by C4, C5 dorsal scapular nerve. This occurs just proximal to the brachial plexus - thus, no trunks or cords are involved in their innervation.

107
Q

Some individuals possess a normal anatomic variant by which their median nerve motor fibers in the hand cross over to join the ulnar nerve. Which of the following is the correct name for this variant?

A

Riche-Cannieu anastomosis involves median motor fibers in the hand crossing over to join the ulnar nerve; classically this can cause the ulnar nerve to supply motor control to the entire hand. The Martin-Gruber anastomosis is another anatomic variant involving median nerve fibers crossing over in the forearm to innervate the ulnar nerve and supply some ulnar nerve muscles in the hand. “Kertley-Thompson anastomosis” and “Shelley-Webster anastomosis” are fictional. Perhaps Kertley and Thompson are indeed out there, searching for their anastomosis of fame

108
Q

During an EMG, you decide to stimulate the ulnar nerve at the wrist and record the CMAP (compound muscle action potential) over the first dorsal interosseous (FDI), whose amplitude appears normal. You then stimulate the ulnar nerve at the elbow and notice a significantly decreased amplitude compared to the wrist, even when adjusting for the patient’s skin conditions and stimulator placement. You decide to leave the recording electrode in place on the FDI. Which of the following is the next best step?

A

Stimulate the median nerve at the elbow
- The Martin-Gruber anastomosis involves median nerve fibers in the forearm crossing over and joining the ulnar nerve in the forearm. This means essentially that the median nerve innervates the abductor digiti minimi (ADM) and first dorsal interosseous (FDI), which are ulnar nerve muscles, in addition to innervating its own classic median nerve muscles (abductor pollicis brevis - APB, etc.). The ulnar nerve CMAP (compound muscle action potential) when stimulating at the elbow will show a low amplitude when recording over a distal ulnar muscle such as FDI. The ulnar CMAP will be “repaired”/normal if you stimulate the ulnar nerve at the wrist. Thus, it appears that there is a conduction block somewhere in the forearm when there actually is not (ulnar nerve conduction block in the mid to distal forearm is also very uncommon). When you stimulate the ulnar nerve at the wrist, at this point the median nerve fibers have finally joined the ulnar nerve, so you are finally stimulating all the motor axons that are supplying the ulnar nerve hand muscles. When you stimulate the ulnar nerve at the elbow, you are not stimulating the median nerve fibers that have yet to contribute to the ulnar innervations, thus you only generate part of the full amplitude, which is what makes it look like there is a conduction block in the ulnar nerve, when in reality there is not. When suspecting a MGA, the physician should perform further testing by recording over the ADM or FDI and stimulating the median nerve at the elbow to see if a sizeable CMAP can be generated (this will not be the full “normal” CMAP, just a small “bump” that represents the small portion of median nerve axons that are destined to cross over and supply the ADM, whereas the lion’s share of median nerve fibers will innervate the classic median nerve muscles and thus will not produce any signal here, as we are recording over the FDI, an ulnar nerve muscle). If MGA is present, the small CMAP amplitude you generate with this will “add up” with the proximal ulnar CMAP to create a full normal-amplitude CMAP, just like the normal CMAP when stimulating the ulnar nerve at the wrist.

109
Q

During an upper limb EMG, you notice prolonged SNAP latency to digit 5, dorsal ulnar cutaneous nerve, and medial antebrachial cutaneous nerve (MAC). Which of the following diagnoses do these findings indicate?

A

Medial cord plexopathy

110
Q

A patient attempts to place their hand in their pocket, but their 5th digit sticks out of the pocket while the remaining fingers dive into the pocket as intended. You are an astute physiatrist and notice this immediately. You tell the patient that they have a positive ______ sign.

A

Wartenberg
- The Wartenberg sign is described, indicating ulnar neuropathy (at the wrist or elbow). The Froment sign is described as the patient attempting to prevent a piece of paper from being pulled out of their grip, by gripping a piece of paper between their index finger and their thumb with the thumb remaining extended the entire time; a positive Froment sign will show the paper slip out easily due to ulnar nerve muscle weakness, or the patient compensating by flexing their thumb’s interphalangeal joint (via flexor pollicis longus - anterior interosseous nerve). The “OK” sign is performed by asking the patient to flex their thumb interphalangeal joint and flex the distal interphalangeal (DIP) joint of the index finger while extending digits 3-5 to make the classic “OK” sign. In patients with anterior interosseous neuropathy (AIN-opathy), thumb flexion (flexor pollicis longus) and DIP flexion (flexor digitorum profundus) will be affected, as these are both AIN functions. These patients will essentially keep a straight finger and thumb while pinching them together, instead of flexing these joints properly. Note: the Froment sign (ulnar neuropathy) and the “OK” sign (AIN-opathy) are essentially opposite tests of each other! The Allen test is a vascular test not high-yield enough for PM&R physicians to know for boards.

111
Q

A patient’s left upper limb EMG/NCS results are as follows. Routine median, ulnar, and radial sensory nerve conduction studies are normal. EMG reveals decreased recruitment and 2+ positive sharp waves in flexor carpi radialis, triceps, extensor carpi radialis longus, flexor digitorum superficialis, but normal results in deltoid, biceps, brachioradialis, abductor pollicis brevis, and first dorsal interosseous. Which of the following is the most likely diagnosis?

A

C7 radiculopathy
- All muscles with C7 innervation are affected here, whereas all those without C7 innervation are spared. Thus, C7 radiculopathy is the best answer.

112
Q

Which of the following can cause an ulnar neuropathy?

A

“Arcade of Struthers” is correct. The Arcade of Struthers is a piece of fascia connecting the brachialis to the triceps, and the ulnar nerve can become entrapped here. The bicipital aponeurosis and Ligament of Struthers cause median neuropathy at the elbow. The Arcade of Frohse causes posterior interosseous neuropathy.

113
Q

Raising the low frequency filter during NCS (nerve conduction studies) will cause which of the following?

A

Shortened peak latency
- Raising the low frequency filter will shorten the peak latency and decrease the amplitude.

114
Q

How should a PM&R physician define ephaptic transmission?

A

A way of firing a motor unit that has undergone a denervation-reinnervation-denervation cycle
- A CRD (complex repetitive discharge) is an involuntary discharge that is very wide and serrated (complex in appearance, and repetitive in firing) and occurs due to a motor unit being denervated and then reinnervated by another motor neuron, which itself then becomes denervated. Ephaptic transmission is the process by which these muscle fibers all fire regularly together. CRDs are seen in chronic radiculopathy, anterior horn cell disease, and some normal patients.

115
Q

Neurogenic thoracic outlet syndrome can be most accurately described as which of the following statements?

A

Lower trunk brachial plexopathy
- Neurogenic TOS is essentially a lower trunk brachial plexopathy (involving C8, T1 nerve roots).

116
Q

A 71 year-old male survives sepsis due to pneumonia. You are consulted to perform an inpatient EMG due to severe weakness reported. The patient complains of weakness in both upper and lower limbs. He denies numbness, tingling, or bowel/bladder dysfunction. NCS reveals normal sensory nerve action potentials (SNAPs) of median, ulnar, radial, superficial fibular, and sural nerves on the left side of the body. There is decreased amplitude of median, ulnar, tibial, and fibular nerve compound muscle action potentials (CMAPs). EMG reveals increased recruitment of abductor pollicis brevis, first dorsal interosseous, deltoid, rectus femoris, gastrocnemius, and tibialis anterior. Which of the following is the most likely diagnosis?

A

Critical illness myopathy (CIM) presents classically as proximal and distal weakness in a critical illness setting (e.g. sepsis/SIRS) with normal sensation and normal SNAPs, and short-duration, small amplitude, early recruitment motor units on EMG - a classic myopathic EMG pattern. SNAPs are normal in CIM, as the nerves are not affected, in contrast to critical illness neuropathy, which will show SNAP abnormalities and a “neuropathic” recruitment pattern (decreased recruitment). Guillain-Barre syndrome typically presents with sensory abnormalities, gradual onset weakness, and recent history of GI infection or upper respiratory tract infection. There is also increased temporal dispersion and conduction block in Guillain-Barre syndrome. Steroid myopathy is due to corticosteroid usage, and typically demonstrates a normal EMG, as it affects type 2 muscle fibers (EMG essentially only examines type 1 muscle fibers).

117
Q

In a Klumpke palsy, which of the following electrodiagnostic findings would most likely be present? SNAP: sensory nerve action potential. PIP: proximal interphalangeal joint.

A

Normal median nerve SNAP to the thumb

118
Q

Steroid myopathy causes which of the following muscle abnormalities?

A

Steroid myopathy causes atrophy of type II muscle fibers (fast-twitch, glycolytic). T

119
Q

A patient’s left upper limb EMG/NCS results are as follows. Routine median, ulnar, and radial sensory nerve conduction studies are normal except for reduced amplitude median SNAP. to digit 2. The lateral antebrachial cutaneous nerve SNAP (sensory nerve action potential) shows decreased amplitude when compared to the contralateral side. EMG reveals decreased recruitment and 1+ positive sharp waves in biceps and brachialis, but normal deltoid, triceps, brachioradialis, extensor indicis proprius, flexor digitorum profundus, abductor pollicis brevis, and first dorsal interosseous. Which of the following muscles would also be expected to demonstrate EMG abnormalities in this patient?

A

Pronator teres
- This patient suffers from a lateral cord brachial plexopathy, which would affect biceps, brachialis, coracobrachialis, pronator teres, flexor carpi radialis, flexor digitorum superficialis, pronator quadratus, flexor pollicis longus, pectoralis major, and lateral antebrachial cutaneous nerve, and may also be expected to affect median nerve SNAP, but not any of the other answer choices listed here.

120
Q

When compared to a concentric needle electrode, a monopolar needle electrode can be described as which of the following?

A

Broader “listening” area
- Monopolar needles achieve a broader listening area (“listening” to the electrical activity of muscles) than concentric needles, but require a separate surface reference electrode on the skin

121
Q

A 72 year-old male is currently being treated in the ICU for urinary tract infection (UTI) leading to sepsis. Having survived the infection, the patient is ordered physical therapy (PT). However, the patient is unable to participate in PT, citing profound weakness and numbness/tingling. He denies a history of this problem. You are called to perform an EMG/NCS to discover the cause of the patient’s symptoms. You decide to study the right hemibody. NCS reveals normal latencies, but decreased amplitude of median, ulnar, radial, tibial, and fibular SNAPs and CMAPs (sensory nerve and compound motor action potentials). EMG reveals long-duration, large amplitude motor units with decreased recruitment and 2+ fibrillations extensor digitorum brevis, abductor hallucis, tibialis anterior, and first dorsal interosseous. Which of the following is the most likely diagnosis?

A

Critical illness neuropathy
- This patient presents with critical illness neuropathy (CIN). It usually involves a history of a critical illness, typically a sepsis/SIRS scenario, and as the patient recovers from the medical aspects of this, they begin to complain of new-onset profound weakness with numbness/tingling. CIN is an axonal sensorimotor polyneuropathy; thus, it will decrease the amplitudes of both SNAPs and CMAPs while leaving the latency/conduction velocity largely intact. EMG shows a neuropathic recruitment pattern (long-duration, large amplitude units with decreased recruitment; the axonal loss causes the active denervation reflected as fibrillations and positive sharp waves). Critical illness myopathy presents classically as proximal>distal weakness in a critical illness setting with normal sensation and normal SNAPs, and short-duration, small amplitude, early recruitment motor units on EMG. Diabetic PN manifests chronically in patients with diabetes, not acutely in the ICU setting. CIDP presents much more gradually as well.

122
Q

During an EMG, a patient is found to have active denervation of the pronator teres and abductor pollicis brevis. Which of the following statements is most likely to be true?

A

This patient has median neuropathy at the elbow. Thus, flexor pollicis longus (FPL) will be affected and potentially weak, making “thumb flexion weakness” the correct choice. Wrist flexion with medial deviation weakness, and DIP flexion weakness to digits 4 and 5 represent ulnar nerve muscle functions (flexor carpi ulnaris and flexor digitorum profundus to digits 4 and 5), and wrist extension weakness represents a radial nerve muscle function. Remember that medial deviation of the wrist = ulnar deviation with respect to conventional anatomic position.

123
Q

You are performing an EMG on a patient with hand numbness. NCS reveals reduced amplitude of the SNAP (sensory nerve action potential) to digit 2 and the CMAP to the abductor pollicis brevis (APB). The SNAP to digit 5 and the CMAP to the abductor digiti minimi are normal. Needle EMG reveals 1+ fibrillations in the APB and flexor carpi radialis (FCR). The remaining muscles, including deltoid, brachioradialis, flexor carpi ulnaris, first dorsal interosseus, and cervical paraspinals, are normal. What is the most likely etiology of this condition?

A

Compression underneath the ligament of Struthers
-This case represents findings of median neuropathy at the elbow (due to active denervation observed in the FCR, a median nerve muscle located at the elbow). This can be caused by compression of the median nerve by a tight pronator teres muscle, bicipital aponeurosis, or ligament of Struthers, all in the elbow. The Arcade of Struthers and the two heads of the FCU can compress the ulnar nerve and cause ulnar neuropathy at the elbow. The normal ulnar nerve studies in this case rule out these answer choices. Compression of the median nerve under the flexor retinaculum causes median neuropathy at the wrist, but the +1 fibs and sharps in the FCR rule out median neuropathy at the wrist as the most likely cause of this patient’s symptoms.

124
Q

During nerve conduction studies, the optimal upper limb temperature is which of the following?

A

32 degrees Celsius

125
Q

Which of the following most accurately describes botulinum toxin’s mechanism of action?

A

Botulinum toxin’s mechanism of action is the inhibition of synaptic vesicles (SNAP-25, syntaxin, synaptobrevin) from releasing acetylcholine (ACh) into the synaptic cleft, thus preventing muscle contraction via lack of ACh stimulus.

126
Q

Which of the following muscles is NOT innervated by the sciatic nerve (tibial division)?

A

Short head of biceps femoris
- Semimembranosus, semitendinosus, and the long head of biceps femoris are all innervated by the sciatic nerve (tibial division). The short head of the biceps femoris is innervated by the sciatic nerve (common fibular division).

127
Q

A 54 year-old male presents with right lateral thigh burning pain with numbness and tingling for the past 3 months. On exam, he is able to trace an ovoid patch on his anterolateral thigh where the abnormal sensations exist. Strength and reflexes are intact. He denies bowel or bladder dysfunction. Which of the following historical findings is most likely present on further questioning?

A

Wearing tight belts
-This patient presents with lateral femoral cutaneous neuropathy (LFCN-opathy, meralgia paresthetica), which is entrapment/compression of this nerve typically as it passes underneath the inguinal ligament. It innervates an ovoid patch of skin on the anterolateral thigh, but does not innervate muscles; thus there will be no muscle weakness, and routine EMG/NCS will be normal. LFCN SNAP will be abnormal, and this should be compared to the unaffected side. LFCN-opathy is associated with diabetes, wearing tight belts/underwear, obesity, and rapid weight changes. It is usually self-limiting but can be treated with physical therapy, neuropathic pain medication (e.g. gabapentin), LFCN block or hydrodissection with ultrasound guidance, or surgical decompression and release.

128
Q

While performing an EMG, you are listening to the right deltoid muscle at rest. You hear a regular sound that sounds like raindrops falling onto a tin roof. What is the most likely clinical implication for this muscle?

A

Active denervation
- The “rain drops on a tin roof” sound, when occurring at regular intervals while the muscle is at rest, is classic for fibrillations. Fibrillations indicate active denervation.

129
Q

In a case of axonotmesis of the median nerve at the wrist on EMG/NCS, 1 month after injury you detect decreased CMAP amplitude proximally and distally to the site of injury in addition to decreased recruitment. 2 years later, which of the following findings would you reasonably expect to discover?

A

Normal CMAP amplitude proximally and distally
- In axonotmesis, even though the axons have died due to crush/stretch injury (leading to decreased CMAP distally and proximally) the epineurium is still intact, which will serve as a guide path for the axons to regenerate along and ultimately find their target muscle fibers again. Thus, months to years later, you may detect a repaired, normal CMAP due to axonal regeneration.

130
Q

During a blink reflex study, you recall that the Vs nucleus is located within which of the following structures?

A

Medulla
- The blink reflex study tests CN V and CN VII in the brainstem and peripherally. CN V has two nuclei being tested (Vm and Vs). The Vm nucleus lies within the pons. It accepts CN V input from the face and conducts it to the ipsilateral CN VII nucleus, causing an ipsilateral blink (R1 response). It also simultaneously conducts the impulse to the medulla where the Vs nucleus lies. Vs accepts the impulse from Vm and sends the impulse to the bilateral CN VII nuclei, which then causes a bilateral blink via the orbicularis oculi muscles (R2 response).

131
Q

During an EMG study, you notice on the screen that with forceful muscle contraction by the patient, there are 2 motor units displayed: one firing at 40 Hz, and the other firing at 50 Hz. Which of the following is the most likely diagnosis?

A

Post-polio syndrome
- This question describes a decreased recruitment pattern, which can be found in cases of conduction block or axonal loss: essentially neuropathies. This is sometimes called a neuropathic recruitment pattern for this reason, as decreased recruitment is generally not seen in myopathies (the remaining answer choices). Post-polio syndrome is the death of anterior horn cells due to “burning out” over time, leaving the patient with few remaining active motor units, which leads to decreased recruitment and, thus, increased firing rate of existing motor units.

132
Q

A 26 year-old male falls on his outstretched hand and sustains a right humeral midshaft fracture. If you were to find weakness in this patient’s right upper limb, which muscle would be most likely to be weak following this fracture?

A

Extensor indicis proprius
- Midshaft humeral fractures raise suspicion for radial nerve injury in the spiral groove. The triceps and anconeus are already innervated at this point, so we must look for any distal radial nerve-innervated muscle. The extensor indicis proprius is the only muscle that fits this category, as it has posterior interosseous nerve innervation which itself is a branch of the radial nerve.

133
Q

Which of the following muscles has the highest innervation ratio?

A

Gluteus maximus
- Innervation ratio refers to the number of muscle fibers innervated by one alpha motor neuron. Remember: “huge” muscles have “huge” innervation ratios. IR = #muscle fibers per motor neuron. In small muscles that require fine control, it makes sense that each muscle fiber “gets more attention” from its alpha motor neuron in order to achieve fine manipulation/control/coordination.

134
Q

A 40-year-old female presents with ptosis, visual changes, and waxing and waning fatigue. Repetitive nerve stimulation shows a decremental response. Which of the following is the most sensitive test to confirm the suspected diagnosis?

A

Single fiber electromyography
- This patient likely has myasthenia gravis (MG), and single fiber electromyography (SF-EMG) is the most sensitive test in diagnosing MG. Antibodies can be useful and sensitive in diagnosing MG. However, anti-MuSK antibodies are often negative in patients with MG (anti-AChR antibodies are more commonly found).

135
Q

A 42 year-old male presents for a left upper limb EMG/NCS. During the study you detect prolonged latency of the sensory nerve action potential (SNAP) to the anatomic snuffbox, with normal SNAPs to digits 2 and 5. Needle EMG reveals decreased recruitment and 2+ fibrillations in brachioradialis and extensor indicis proprius (EIP), with normal EMG of deltoid, biceps brachii, triceps, pronator teres, first dorsal interosseous, and abductor pollicis brevis. Which of the following is the most likely cause of the patient’s symptoms?

A

Midshaft humerus fracture
- This patient presents with EMG/NCS findings localizing to the radial nerve at or just proximal to the elbow. Midshaft humerus fractures classically produce radial neuropathy that spares the triceps and anconeus (as these muscles are innervated prior to the humeral midshaft area - the spiral groove). Improper crutch use can cause a proximal radial neuropathy, but that would demonstrate EMG abnormalities in the triceps/anconeus, and the triceps is normal in this patient’s study. Compression by the Arcade of Frohse can cause posterior interosseous neuropathy (PIN-opathy) - a pure motor neuropathy that would spare brachioradialis (not a PIN-innervated muscle) and show normal radial SNAPs (our radial SNAP in this patient’s study is abnormal). Finally, the ligament of Struthers can cause median neuropathy at the elbow, and our median nerve studies are normal in this patient.

136
Q

Years following a neurotmesis injury in the forearm, what is the most reasonable expectation on EMG/NCS?

A

Absent CMAP
- Neurotmesis is the complete transection of a nerve. Thus, axons have no connective tissue nerve path through which they may find their former muscle fibers, and the CMAP most likely will not be normal or partially repaired.

137
Q

A 56 year-old male with a history of cancer within the posterior triangle of the neck presents to you with 5 months of left shoulder pain. On exam, his left scapula rests more laterally than his right scapula. This problem is most likely the result of injury to which of the following nerves?

A

Spinal accessory
- A laterally winged scapula (as in this patient) indicates upper/middle trapezius muscle weakness, which is most commonly due to spinal accessory neuropathy. The spinal accessory nerve passes through the posterior triangle of the neck and can be impinged by structures or abnormal masses in this area. The long thoracic nerve innervates the serratus anterior, and injury to this nerve can cause a medially winged scapula (not laterally winged). Injury to the thoracodorsal nerve, which innervates the latissimus dorsi, does not cause a winged scapula. Injury to the dorsal scapular nerve (which innervates the rhomboids) is rare, and would potentially also be a cause of laterally winged scapula. However, the most likely answer remains the spinal accessory nerve.

138
Q

The quadriceps femoris is innervated by which of the following nerve root groups?

A

L2, L3, L4
- The quadriceps are innervated by L2, L3, L4 nerve roots. The femoral nerve is the peripheral nerve that innervates the quadriceps.

139
Q

A 23 year-old male sustained an injury while playing soccer 2 weeks ago. He presents for an EMG evaluation due to foot drop. X-rays of the right lower limb are negative. On exam, he demonstrates intact lower limb strength except for 3+/5 right ankle dorsiflexion. Tinel at the fibular head is positive for reproduction of numbness and tingling he is experiencing. Nerve conduction studies demonstrate normal sural sensory nerve action potential (SNAP), prolonged latency of the superficial fibular SNAP, and normal compound muscle action potential (CMAP) of the tibialis anterior (TA) when stimulating below the fibular head. However, when stimulating within the popliteal fossa, the TA CMAP amplitude drops 50% compared to the below fibular head stimulation site. Needle EMG of the TA and extensor digitorum brevis (EDB) reveals decreased recruitment. Needle EMG of remaining muscles is normal. Which of the following is the most appropriate response to these findings?

A

Prognosis for recovery is good
- This patient’s results demonstrate neurapraxia of the common fibular nerve, likely due to some physical injury sustained during the soccer match. Neurapraxia can be differentiated from axonotmesis (axon loss due to crush/stretch injury) and neurotmesis (axon loss due to complete nerve transection) due to the normal CMAPs present below the level of injury at 2 weeks out from injury. There is also the lack of active denervation (fibrillations and positive sharp waves) that suggests neurapraxia rather than axon loss. Neurapraxia, you recall, manifests as a conduction block, as in this case (normal amplitude CMAP distal to the lesion, decreased amplitude CMAP proximal to the lesion). Neurapraxia/conduction block represents a focal demyelination event; thus the axons are intact, and all that is required is remyelination for the CMAP, recruitment, and patient’s strength to recover. If this were axon loss, the CMAP amplitude would also be decreased distally at this point (2 weeks; recall that within ~10 days Wallerian degeneration is complete for both sensory and motor fibers); thus, the intact distal CMAP suggests the axons are alive and healthy, and just awaiting remyelination; thus, this patient’s prognosis for spontaneous recovery is good. Thus, surgery would be inappropriate, MRI is unnecessary, and US-guided intervention is unnecessary.

140
Q

You are performing an EMG and decide to advance the needle into a muscle at rest when you hear what sounds like a “divebomber”. Which of the following is the most likely diagnosis?

A

All of the listed choices are correct
- divebomber sound is classic for myotonic discharges. This can be seen on the screen as a steadily decreasing amplitude as the muscle fiber continues to fire. Essentially anything with “myotonia” in its name can demonstrate myotonic discharges on EMG, as can hyperkalemic periodic paralysis and acid maltase deficiency.

141
Q

Which of the following nerves innervates the muscles of the deep posterior compartment of the leg?

A

The deep posterior leg compartment contains the tibialis posterior, flexor digitorum longus, and flexor hallucis longus, which are all innervated by the tibial nerve.

142
Q

A 67 year-old female with a history of facial trauma presents with complaints of crying every time she eats. She denies any actual emotional disturbances. What is the most likely etiology of this problem?

A

Synkinesis
- Synkinesis is the inappropriate regeneration of the facial nerve after it is damaged, leading to the nerve reinnervating muscles inappropriately in such a fashion that the patient may end up doing two “facial nerve” activities at once when the patient only means to do one of these. For example, blinking and moving their mouth at the same time, or salivating whenever they cry.

143
Q

You are performing an EMG on a patient with hand numbness. NCS reveals non-recordable sensory nerve action potential (SNAP) to digit 2, and normal SNAP to digit 5. The compound motor action potential (CMAP) to the abductor pollicis brevis (APB) demonstrates decreased amplitude and prolonged latency. Needle EMG to the APB reveals 1+ fibrillations and positive sharp waves and mildly decreased recruitment. EMG of the remaining muscles, including the pronator teres, is normal. Which of the following is the next best step?

A

Hand surgeon referral
- This patient with absent median nerve SNAP, reduced median CMAP amplitude, prolonged motor latency, and active denervation to the APB (median nerve-innervated muscle, active denervation as evidenced by fibrillations and positive sharp waves on EMG) should be referred to a hand surgeon for carpal tunnel release. This case represents severe CTS due to active denervation and decreased CMAP amplitude, together representing ongoing axon loss. (moderate to severe cases are usually most appropriate for surgeon referral). Wrist bracing and ibuprofen with EMG monitoring is appropriate for mild cases (prolonged SNAP latency only). Injection is appropriate for mild-moderate cases that fail to improve with conservative treatment (moderate CTS: prolonged SNAP and CMAP latency but normal CMAP amplitude).

144
Q

During a blink reflex study, the input and output are via which cranial nerves, respectively?

A

5, 7
- The blink reflex study tests CN V and CN VII in the brainstem and peripherally. CN V has two nuclei being tested (Vm and Vs). The Vm nucleus lies within the pons. It accepts CN V input from the face and conducts it to the ipsilateral CN VII nucleus, causing an ipsilateral blink (R1 response). It also simultaneously conducts the impulse to the medulla where the Vs nucleus lies. Vs accepts the impulse from Vm and sends the impulse to the bilateral CN VII nuclei, which then causes a bilateral blink via the orbicularis oculi muscles (R2 response).

145
Q

During an EMG/NCS study, routine sensory nerve action potentials (SNAPs) and compound muscle action potentials (CMAPs) are normal. Needle EMG reveals abnormal spontaneous activity in the infraspinatus, and normal EMG of the cervical paraspinals, deltoid, supraspinatus, triceps, pronator teres, abductor pollicis brevis, and first dorsal interosseous. Which of the following is the most likely diagnosis?

A

Suprascapular neuropathy distal to the spinoglenoid notch
- The suprascapular nerve innervates the supraspinatus, then passes through the spinoglenoid notch to innervate the infraspinatus. Thus, if injury occurs distal to the spinoglenoid notch, only the infraspinatus will show abnormalities, as the supraspinatus is already innervated at that point. If the injury to the suprascapular nerve occurs proximal to the spinoglenoid notch, then both supraspinatus and infraspinatus should show abnormalities. Upper trunk plexopathy would also show abnormalities in the deltoid (normal in our patient’s study). C5 radiculopathy would also show abnormalities in the deltoid and cervical paraspinals (normal in our patient’s study).

146
Q

The purpose of EMG/nerve conduction studies is to diagnose disorders of which of the following?

A

Peripheral nervous system

147
Q

The supraspinatus and infraspinatus are innervated by which of the following pathways?

A

C5, C6, upper trunk, suprascapular nerve

148
Q

The lateral antebrachial cutaneous nerve (LAC) arises in which of the following ways?

A

The LAC is actually just the terminal portion of the musculocutaneous nerve, after it has innervated all its muscles.

149
Q

Normal upper and lower extremity conduction velocities are at least which of the following, respectively, in meters per second?

A

50, 40

150
Q

Which of the following is the most common non-traumatic cause of ulnar neuropathy at the wrist?

A

Ganglion cysts comprise the majority of cases of ulnar neuropathy at the wrist not due to trauma. Bicyclists are a common patient demographic for ganglion cysts causing ulnar neuropathy at the wrist.

151
Q

The “OK” sign is a test for which of the following nerves?

A

Anterior interosseous nerve
- Note: the Froment sign (ulnar neuropathy) and the “OK” sign (AIN-opathy) are essentially opposite tests of each other!

152
Q

You are assessing a patient for an L5 radiculopathy. As a sharp physiatrist, you decide to test the strength of the extensor hallucis longus (EHL), which is 4/5. Which of the following functions would be most useful to test next?

A

Ankle inversion
- The only function here which is performed by an L5-innervated muscle is ankle inversion (tibialis posterior - L5, S1; sciatic nerve; tibial nerve). Knee extension is performed by the quadriceps (L2, L3, L4). Plantarflexion is controlled by the gastrocnemius-soleus complex (S1, S2; sciatic nerve; tibial nerve). Thigh adduction is controlled by the adductor muscles (L2, L3, L4; obturator nerve).

152
Q
A
153
Q

A patient’s right upper limb EMG/NCS results are as follows. Routine median, ulnar, and radial sensory nerve conduction studies are normal. EMG reveals decreased recruitment and 3+ fibrillations in supraspinatus, deltoid, biceps, and brachioradialis. EMG of triceps, pronator teres, extensor indicis proprius, abductor pollicis brevis, and first dorsal interosseous is normal. Which of the following is the most likely diagnosis?

A

C5 radiculopathy
- C5 radiculopathy is the best answer. All upper limb muscles receiving C5 innervation are affected (infraspinatus, brachialis, and teres minor are untested, but could be expected to show changes as well). Muscles with C5-C6 innervation are affected (supraspinatus, deltoid, biceps, and brachioradialis), but those without any C5 innervation are spared (all remaining muscles). Pronator teres has C6-C7 innervation, thus it is spared. Upper trunk plexopathy would be expected to show median sensory abnormalities on NCS, as well as pronator teres and possibly triceps being affected. Lateral cord plexopathy would affect biceps, brachialis, coracobrachialis, pronator teres, flexor carpi radialis, flexor digitorum superficialis, pronator quadratus, flexor pollicis longus, pectoralis major, and lateral antebrachial cutaneous nerve. Remember, sensory nerve action potentials are always normal in a purely radiculopathic process. Please refer to the innervation chart at the beginning of the Upper and Lower Extremity Peripheral Nervous System Diseases chapters for a detailed innervation guide.

154
Q

A 50-year-old male presents with left foot drop. EMG shows positive sharp waves and fibrillation potentials in the tibialis anterior and extensor hallucis longus muscles. No denervation potentials are observed in the lumbar paraspinal muscles, gluteus medius, semimembranosus, gastrocnemius, and peroneus longus. What is the most likely diagnosis?

A

Deep peroneal/fibular neuropathy
- Deep peroneal neuropathy most likely produces the findings in this case. The tibialis anterior and extensor hallucis longus receive their innervation from the deep branch of the peroneal nerve, whereas the peroneus longus receives its innervation from the superficial branch of this nerve (making common peroneal neuropathy less likely). Other L5-innervated and sciatic-innervated muscles within this question stem are within normal limits, making deep peroneal/fibular neuropathy the most likely diagnosis.

155
Q

A 60-year-old female presents with weakness in her left hand. NCS shows reduced motor amplitude in the ulnar nerve at the wrist without a significant drop in velocity across the elbow. You suspect an ulnar neuropathy at the wrist. What additional study is helpful in distinguishing ulnar neuropathy at the wrist versus the elbow, and what is the expected result in this patient?

A

Dorsal ulnar cutaneous – normal
- The dorsal ulnar cutaneous nerve is a branch of the ulnar nerve that branches off the ulnar nerve proximal to Guyon’s canal and provides innervation to the dorsal aspect of the ulnar-innervated hand. Because it branches proximally to the wrist, it is expected to be normal in an entrapment at the wrist but would be affected by an entrapment at the elbow. The medial antebrachial cutaneous nerve is a direct branch of the medial cord of the brachial plexus and provides sensory innervation to the medial forearm. It is not affected in a purely ulnar neuropathy.

156
Q

During electrodiagnostic studies, the A-wave can be studied. Which of the following best describes the A-wave?

A

A-waves show there has been previous reinnervation involving the nerve to the muscle being studied
- A-waves (also called the axon reflex - not a true reflex) are a very predictable, stable waveform that shows up somewhere between the F-wave and the direct motor response when recording F-waves from a muscle. It is the exact same waveform with every stimulation (same latency and amplitude). If present, A-waves indicate that there has been previous reinnervation involving the nerve to the muscle being studied

157
Q

A 55-year-old female presents with progressive weakness and fasciculations in her upper limbs. EMG demonstrated fibrillations, positive sharp waves, and fasciculations in multiple muscles of the upper and lower limbs bilaterally. You suspect a diagnosis of ALS. Assuming no other nerve injuries, what finding would you expect on nerve conduction studies?

A

Normal nerve conduction studies
- ALS classically manifests in electrodiagnostic studies with diffuse fasciculations, fibrillations, and positive sharp waves. NCS is typically normal in ALS, distinguishing it from similar conditions such as multifocal motor neuropathy.

158
Q

A 40 year-old female develops 4 days of progressive lower limb weakness. She has a history of recent upper respiratory tract infection. Bilateral lower limb EMG/NCS reveals normal sural nerve SNAPs (sensory nerve action potentials), but prolonged latency of bilateral superficial fibular nerve SNAPs. The bilateral CMAPs to the EDB and AH (compound muscle action potentials to the extensor digitorum brevis and abductor hallucis) show prolonged latency but normal amplitude. EMG reveals decreased recruitment in the bilateral EDB, AH, tibialis anterior, gastrocnemius, and semimembranosus, but normal activity in rectus femoris and adductor longus. Which of the following additional EMG/NCS findings would most likely be discovered in this patient?

A

Absent F-waves
- This patient presents with Guillain-Barre Syndrome (GBS), also referred to as AIDP (acute inflammatory demyelinating polyradiculopathy). GBS is caused by a recent infection (typically GI or URI) causing your immune system to confuse a foreign protein with a protein on your own myelin. Thus, your body destroys your own peripheral nerve myelin (demyelinating the nerves and prolonging the latency). The first EDX finding of GBS is delayed or absent F-waves. NCS reveals prolonged latency and sometimes reduced amplitude of SNAPs with sural nerve sparing, as the sural nerve is larger with more myelin than other peripheral sensory nerves, and thus is less affected than the other nerves. CMAPs show prolonged latency, decreased conduction velocity (CV), and typically normal amplitude (this is a demyelinating disease primarily, not axonal loss). EMG shows decreased recruitment but otherwise there should be no active denervation usually, but this can occur as a secondary feature of GBS weeks after onset. Treatment involves PT and early plasmapheresis/IVIG. Abnormal/increased temporal dispersion is also a feature of this disease on NCS, due to the widespread and more uniform demyelination of nerves taking place. The presence of A-waves indicates that some reinnervation has taken place in the past, and would not be a criterion in diagnosing GBS.

158
Q

A 48 year-old male develops right thigh pain and weakness. Strength testing of hip flexion, knee extension, dorsiflexion, great toe extension, and plantarflexion is normal. EMG/NCS reveals normal sensory nerve action potentials (SNAPs) of sural and superficial fibular nerves, and normal compound muscle action potentials (CMAPs) to the extensor digitorum brevis and abductor hallucis. Needle EMG reveals decreased recruitment and 2+ fibrillations in adductor longus, and normal activity in abductor hallucis, extensor digitorum brevis, gastrocnemius, rectus femoris, tensor fascia lata, and lumbar paraspinals. Which of the following is the most likely diagnosis?

A

Obturator neuropathy
- patient’s findings suggest obturator neuropathy, typically due to trauma such as a pelvic fracture. Femoral neuropathy and L3/L4 radiculopathy would show abnormal activity in the quadriceps, which are spared in this patient. This patient would also demonstrate thigh adduction weakness, which was not evaluated in this patient.

158
Q

During nerve conduction studies, the act of stimulating a motor nerve in a proximal (antidromic) direction and recording from a distal muscle innervated by this nerve is performed in order to obtain which of the following signals?

A

F Wave
- This question describes how to obtain an F wave. For example, recording over the first dorsal interosseus muscle and antidromically stimulating the ulnar nerve at the wrist will send an action potential antidromically back up, all the way to the anterior horn, which will then depolarize a random population of anterior horn cells, whose own depolarization will then travel back down the axons of the ulnar nerve and will be recorded by the recording electrode over the first dorsal interosseus muscle. Thus, this signal, the F wave, is not actually a true reflex, although it is sometimes called the “F reflex”.

159
Q

During repetitive nerve stimulation (RNS), which of the following percentage decreases in the compound muscle action potential (CMAP) amplitude indicates that a neuromuscular junction (NMJ) disease is present?

A

A greater than 10% decrement in the CMAP amplitude during RNS is a positive finding for NMJ disease (myasthenia gravis, Lambert-Eaton Syndrome, botulism).

160
Q

In pronator teres syndrome, all median nerve-innervated muscles are affected EXCEPT which of the following?

A

Pronator teres
- In PT (pronator teres) syndrome, the median nerve is compressed within the PT muscle, affecting all downstream median nerve-innervated muscles, except the PT itself, which is actually innervated by a more proximal nerve branch that is not involved in the PT’s tight compression of the median nerve. All remaining median nerve and anterior interosseous nerve muscles will be affected, however (the remaining answer choices).

161
Q

Which of the following muscles receives its innervation from all roots (C5-T1) of the brachial plexus?

A

Pectoralis major
- pectoralis major is innervated by C5-T1 roots of the brachial plexus. Triceps and latissimus dorsi are innervated by C6, C7, C8 roots. FDP is innervated by C7, C8, T1 roots.

162
Q

Which of the following innervation pathways belongs to the extensor digitorum brevis (EDB)?

A

L4, L5, S1; sciatic nerve; deep fibular nerve

163
Q

A 60-year-old female presents with weakness in her left hand. NCS shows reduced motor amplitude in the ulnar nerve at the wrist without a significant drop in velocity across the elbow. You suspect an ulnar neuropathy at the wrist. What additional study is helpful in distinguishing ulnar neuropathy at the wrist versus the elbow, and what is the expected result in this patient?

A

Dorsal ulnar cutaneous – normal
- The dorsal ulnar cutaneous nerve is a branch of the ulnar nerve that branches off the ulnar nerve proximal to Guyon’s canal and provides innervation to the dorsal aspect of the ulnar-innervated hand. Because it branches proximally to the wrist, it is expected to be normal in an entrapment at the wrist but would be affected by an entrapment at the elbow. The medial antebrachial cutaneous nerve is a direct branch of the medial cord of the brachial plexus and provides sensory innervation to the medial forearm. It is not affected in a purely ulnar neuropathy.